Изменится ли объем газа если перекачать его из баллона: Изменится ли (и если да,то как)объем газа,если перекачать его из баллона10 л в баллон 30 литров? а)нет,не изменится б)да,увеличится

Содержание

Контрольные работы по физике «Первоначальные сведения о строении вещества» 7 класс

Контрольная работа №1 по теме

«Первоначальные сведения о строении вещества». Физика 7

Вариант 1

Часть А

1. Физическим телом является …:

1) самолет 2) вода 3) метр 4) кипение

2. Веществом является…:

1) килограмм 2) звук 3) алюминий 4) Земля

3. К звуковым явлениям относятся:

1) шар катится 2) слышны раскаты грома 3) снег тает

4) наступает рассвет

4. Определить цену деления

барометра, изображенного на

рисунке в мм рт. ст.

1) 1 мм. рт. ст 2) 10 мм. рт. ст

3) 5 мм. рт. ст 4) 2 мм. рт. ст

5. Молекулы льда и воды отличаются друг от друга:

1) Количеством атомов 2) Формой

3) Размером

4) Молекулы одного и того же вещества в жидком и в твердом состояниях

одинаковы

6. Явление диффузии доказывает…

1) Только факт существования молекул

2) Только факт движения молекул.

3) Факт существования и движения молекул

4) Факт взаимодействия молекул

7. Между молекулами любого вещества действуют

1) Только силы отталкивания 2) Только силы притяжения

3) Силы притяжения и отталкивания 4) Не действуют никакие силы

8. Какое явление служит доказательством того, что между частицами вещества

проявляются силы притяжения:

1) Свинцовые цилиндры слипаются, если их прижать друг к другу свежими

срезами.

2) Сахар растворяется в воде 3) Лед тает в теплом помещении

4) При прохождении тока электрическая лампочка светится

9. Тело, в котором молекулы расположены на больших расстояниях друг

относительно друга, слабо взаимодействуют между собой, движутся хаотически:

1) Газ 2) Твердое тело 3) Жидкость

4) Или твердое тело, или жидкость.

10. Жидкость:

1) Занимает объем всего сосуда 2) Легко поддается сжатию

3) Принимает форму сосуда

4) Имеют кристаллическое строение.

Часть В.

Молекулы плотно упакованы сильно притягиваются. Тест «Молекулярная физика

IV вариант

I- 3

II- 1, 4

III- 3

IV- 2

Тема: Три состояния вещества

I вариант

I. Как расположены молекулы в твёрдых телах и как они движутся?

Молекулы расположены на расстояниях меньших размеров самих молекул и перемещаются свободно относительно друг друга. Молекулы расположены на больших расстояниях друг от друга (по сравнению с размерами молекул) и движутся беспорядочно. Молекулы расположены в строгом порядке и колеблются около определённых положений равновесия.

II. Какие из приведённых ниже свойств принадлежат газам?

Имеют определённый объём Занимают объём всего сосуда Принимают форму сосуда Мало сжимаются Легко поддаются сжатию

III. Изменится ли объём газа, если его перекачать из сосуда вместимостью 1 литр в сосуд вместимостью 2 литра?

IV. Молекулы расположены на больших расстояниях друг от друга (по отношению с размерами молекул), слабо взаимодействуют между собой, движутся хаотически. Какое это тело?

Газ Твёрдое тело Жидкость Такого тела нет

V. В каком состоянии может находиться сталь?

Только в твёрдом состоянии Только в жидком состоянии Только в газообразном Во всех трёх состояниях

Тема: Три состояния вещества

II вариант

I. Как расположены молекулы жидкостей и как они движутся?

Молекулы расположены на расстояниях, соизмеримых с размерами самих молекул, и перемещаются свободно относительно друг друга. Молекулы расположены на больших расстояниях (по сравнению с размерами молекул) друг от друга и движутся беспорядочно. Молекулы расположены в строгом порядке и колеблются около определённых положений равновесия.

II. Какие из приведённых свойств принадлежат газам?

Занимают весь предоставленный им объём Трудно сжимаются Имеют кристаллическое строение Легко сжимаются Не имеют собственной формы

III. В мензурке находится вода объёмом 100 см3. Её переливают в стакан вместимостью 200 см3. Изменится ли объём воды?

IV. Молекулы плотно упакованы, сильно притягиваются друг к другу, каждая молекула колеблется около определённого положения. Какое это тело?

Газ Жидкость Твёрдое тело Таких тел нет

V. В каком состоянии может находиться вода?

Только в жидком состоянии Только в газообразном состоянии Только в твёрдом состоянии Во всех трёх состояниях

Тема: Три состояния вещества

III вариант

I. Как расположены молекулы газов и как они движутся?

Молекулы расположены на расстояниях, меньших размеров самих молекул, и перемещаются свободно относительно друг друга. Молекулы расположены на расстояниях, во много раз больше размеров самих молекул, и движутся беспорядочно. Молекулы расположены в строгом порядке и колеблются около определённых положений.

II. Какие из приведённых свойств принадлежат твёрдым телам?

Трудно изменить форму Занимают весь предоставленный им объём Сохраняют постоянную форму Легко меняют форму Трудно сжимаются

III. Изменится ли объём газа, если его перекачать из баллона вместимостью 20 литров в баллон вместимость.40 литров?

Увеличится в 2 раза Уменьшится в 2 раза Не изменится

IV. Есть ли такое вещество, у которого молекулы расположены на больших расстояниях, сильно притягиваются друг к другу и колеблются около определённых положений?

Газ Жидкость Твёрдое тело Такого вещества не существует

V. В каком состоянии может находиться ртуть?

Только в жидком Только в твёрдом Только в газообразном Во всех трёх состояниях

Тема: Три состояния вещества

IV вариант

I. Ниже указано поведение молекул в твёрдых, жидких и газообразных телах. Что является общим для жидкостей и газов?

То, что молекулы расположены на расстояниях меньших размеров самих молекул и движутся свободно относительно друг друга То, что молекулы расположены на больших расстояниях друг от друга и движутся беспорядочно То, что молекулы движутся беспорядочно друг относительно друга То, что молекулы расположены в строгом порядке и колеблются около определённых положений

II. Какие из указанных свойств принадлежат твёрдым телам?

Имеют определённый объём Занимают объём всего сосуда Принимают форму сосуда Мало сжимаются Легко сжимаются

III. В бутылке находится вода объёмом 0,5 литра. Её переливают в колбу вместимостью 1 литр. Изменится ли объём воды?

Увеличится Уменьшится Не изменится

IV. Молекулы расположены так, что расстояние между ними меньше размеров самих молекул. Они сильно притягиваются друг к другу и перемещаются с места на место. Какое это тело?

Газ Жидкость Твёрдое тело

V. В каком состоянии может находиться спирт?

Только в твёрдом состоянии Только в жидком состоянии Только в газообразном состоянии Во всех трёх состояниях

Ответы к тестам

I вариант

II — 2 , 5

II вариант

II — 1, 4, 5

III вариант

II — 1, 3, 5

IV вариант

II — 1, 4

Тест по теме «Три состояния вещества». 7 класс. 1 вариант. 1. Какие из приведённых свойств не принадлежат жидкостям? А. Имеют определённый объём. Б. Занимают объём всего сосуда. В. Принимают форму сосуда. Г. Мало сжимаются. 2. Изменится ли объём газа, если его перекачать из сосуда вместимостью 1 л в сосуд вместимостью 2 л? А. Увеличится в 2 раза. Б. Уменьшится в 2 раза. В. Не изменится. 3. Молекулы расположены на больших расстояниях друг от друга, слабо взаимодействуют между собой, движутся хаотически. Какое это тело? А. Газ. Б. Твердое тело. В. Жидкость. Г. Такого тела нет. 4. В каких телах молекулы колеблются около положения равновесия? А. В твёрдых Б. В жидких В. В газообразных. Г. Во всех трех состояниях. 5. В каких телах диффузия происходит быстрее? А. Одинаково во всех телах. Б. В твердых телах. В. В жидкостях. Г. В газах. Тест по теме «Три состояния вещества». 7 класс. 2 вариант.

1. Какие из приведённых свойств не принадлежат газам? А. Занимают весь предоставленный объём. Б. Трудно сжимаются. В. Легко сжимаются. Г. Не имеют собственной формы. 2. В мензурке находится вода объёмом 100 см3. Её переливают в стакан вместимостью 200 см3. Изменится ли объём воды? А. Увеличится в 2 раза. Б. Уменьшится в 2 раза. В. Не изменится. 3. Молекулы плотно упакованы, сильно притягиваются друг к другу, и каждая молекула колеблется около определённой точки. Какое это тело? А. Газ. Б. Твердое тело. В. Жидкость. Г. Такого тела нет. 4. Почему жидкости не имеют формы? А. Молекулы жидкости слабо взаимодействуют друг с другом Б. Молекулы жидкости способны перемещаться с места на место В. Молекулы жидкости не имеют формы Г. Молекулы жидкости двигаются хаотично 5. Молекулы притягиваются друг к другу. Но почему между ними существуют промежутки? А. Они движутся. Б. Очень слабо притягиваются друг к другу. В. При большом сближении отталкиваются друг от друга. Тест по теме «Три состояния вещества». 7 класс. 1 вариант. 1. Какие из приведённых свойств не принадлежат жидкостям? А. Имеют определённый объём. Б. Занимают объём всего сосуда. В. Принимают форму сосуда. Г. Мало сжимаются. 2. Изменится ли объём газа, если его перекачать из сосуда вместимостью 1 л в сосуд вместимостью 2 л? А. Увеличится в 2 раза. Б. Уменьшится в 2 раза. В. Не изменится. 3. Молекулы расположены на больших расстояниях друг от друга, слабо взаимодействуют между собой, движутся хаотически. Какое это тело? А. Газ. Б. Твердое тело. В. Жидкость. Г. Такого тела нет. 4. В каких телах молекулы колеблются около положения равновесия? А. В твёрдых Б. В жидких В. В газообразных. Г. Во всех трех состояниях. 5. В каких телах диффузия происходит быстрее? А. Одинаково во всех телах. Б. В твердых телах. В. В жидкостях. Г. В газах. Тест по теме «Три состояния вещества». 7 класс. 2 вариант. 1. Какие из приведённых свойств не принадлежат газам? А. Занимают весь предоставленный объём. Б. Трудно сжимаются. В. Легко сжимаются.
Г. Не имеют собственной формы. 2. В мензурке находится вода объёмом 100 см3. Её переливают в стакан вместимостью 200 см3. Изменится ли объём воды? А. Увеличится в 2 раза. Б. Уменьшится в 2 раза. В. Не изменится. 3. Молекулы плотно упакованы, сильно притягиваются друг к другу, и каждая молекула колеблется около определённой точки. Какое это тело? А. Газ. Б. Твердое тело. В. Жидкость. Г. Такого тела нет. 4. Почему жидкости не имеют формы? А. Молекулы жидкости слабо взаимодействуют друг с другом Б. Молекулы жидкости способны перемещаться с места на место В. Молекулы жидкости не имеют формы Г. Молекулы жидкости двигаются хаотично 5. Молекулы притягиваются друг к другу. Но почему между ними существуют промежутки? А. Они движутся. Б. Очень слабо притягиваются друг к другу. В. При большом сближении отталкиваются друг от друга. Тест по теме «Три состояния вещества». 7 класс. 1 вариант. 1. Какие из приведённых свойств не принадлежат жидкостям? А. Имеют определённый объём. Б. Занимают объём всего сосуда. В. Принимают форму сосуда. Г. Мало сжимаются. 2. Изменится ли объём газа, если его перекачать из сосуда вместимостью 1 л в сосуд вместимостью 2 л? А. Увеличится в 2 раза. Б. Уменьшится в 2 раза. В. Не изменится. 3. Молекулы расположены на больших расстояниях друг от друга, слабо взаимодействуют между собой, движутся хаотически. Какое это тело? А. Газ. Б. Твердое тело. В. Жидкость. Г. Такого тела нет. 4. В каких телах молекулы колеблются около положения равновесия? А. В твёрдых Б. В жидких В. В газообразных. Г. Во всех трех состояниях. 5. В каких телах диффузия происходит быстрее? А. Одинаково во всех телах. Б. В твердых телах. В. В жидкостях. Г. В газах. Тест по теме «Три состояния вещества». 7 класс. 2 вариант. 1. Какие из приведённых свойств не принадлежат газам? А. Занимают весь предоставленный объём. Б. Трудно сжимаются. В. Легко сжимаются. Г. Не имеют собственной формы. 2. В мензурке находится вода объёмом 100 см3. Её переливают в стакан вместимостью 200 см3. Изменится ли объём воды? А.
Увеличится в 2 раза. Б. Уменьшится в 2 раза. В. Не изменится. 3. Молекулы плотно упакованы, сильно притягиваются друг к другу, и каждая молекула колеблется около определённой точки. Какое это тело? А. Газ. Б. Твердое тело. В. Жидкость. Г. Такого тела нет. 4. Почему жидкости не имеют формы? А. Молекулы жидкости слабо взаимодействуют друг с другом Б. Молекулы жидкости способны перемещаться с места на место В. Молекулы жидкости не имеют формы Г. Молекулы жидкости двигаются хаотично 5. Молекулы притягиваются друг к другу. Но почему между ними существуют промежутки? А. Они движутся. Б. Очень слабо притягиваются друг к другу. В. При большом сближении отталкиваются друг от друга.

Тестовые задания по теме “Молекулярная физика”.

Вариант 1

1.Как расположены молекулы жидкостей и как они движутся?

2. Какие из приведенных свойств принадлежат газам?

А) Занимают весь предоставленный им объем;

Б) трудно сжимаются;

В) имеют кристаллическое строение;

Г) легко сжимаются;

Д) не имеют собственной формы.

3. В мензурке находится вода объёмом 100 см 3 . Её переливают в стакан вместимостью 200 см 3 . Изменится ли объем воды?

А) увеличится;

Б) уменьшится;

В) не изменится.

4. Молекулы плотно упакованы, сильно притягиваются друг к другу, и каждая молекула колеблется около определенного положения. Какое это тело?

А) газ;

Б) жидкость;

В) твердое тело;

Г) таких тел нет.

5. В каком состоянии может находиться вода:

А) только в жидком;

Б) только в газообразном состоянии;

В) только в твердом состоянии;

Г) во всех трех состояниях.

6. Как расположены молекулы газов и как они движутся?

А) молекулы расположены на расстояниях, меньших размеров самих молекул, и перемещаются свободно друг относительно друга;

Б) молекулы расположены на расстояниях, во много раз больших размеров самих молекул, и движутся беспорядочно;

В) молекулы расположены в строгом порядке и колеблются около определенных положений.

7. Какие из приведенных свойств принадлежат твердым телам? (Выберите несколько вариантов)

А) трудно изменить форму;

Б) занимают весь предоставленный им объем;

В) сохраняют постоянную форму;

Г) легко изменяют форму;

Д) трудно сжимаются.

8. Изменится ли объем газа, если его перекачать из баллона вместимостью 20 л в баллон вместимостью 40 л?

А) увеличится в 2 раза;

Б) уменьшится в 2 раза;

В) не изменится.

9. Есть ли такое вещество, у которого молекулы расположены на больших расстояниях, сильно притягиваются друг к другу и колеблются около определенных положений?

А) газ;

Б) жидкость;

В) твердое тело;

Г) такого вещества нет.

10. В каком состоянии может находиться ртуть: твердом, жидком или газообразном?

А) только в жидком;

Б) только в твердом состоянии;

Г) во всех трех состояниях.

11. Условие идеальности газа:

А) диаметр молекул меньше среднего расстояния между этими молекулами;

Б) кинетическая энергия много больше средней потенциальной энергии их взаимодействия на расстоянии большем диаметра молекул;

В) молекулы взаимодействуют со стенками сосуда и между собой упруго;

Г) все утверждения верны.

12. Взаимодействие между молекулами носит характер:

А) притяжения;

Б) отталкивания;

В) отталкивания на малых расстояниях, притяжения – на больших;

Г) притяжения на малых расстояниях, отталкивания — на больших.

13. При конденсации в паре остаются молекулы, обладающие…

Б) наименьшей кинетической энергией

14. Если жидкость находится в равновесии со своим насыщенным паром, то скорость испарения жидкости…

А) больше скорости конденсации пара;

Б) меньше скорости конденсации пара;

В) равна скорости конденсации пара;

Г) равна нулю.

Вариант — 2

1. В сосуде под поршнем находится ненасыщенный пар. Его можно перевести в насыщенный…

А) повышая температуру;

В) увеличивая объем пара;

Г) уменьшая объем пара.

2. Плотность насыщенного пара…

А) с увеличением давления – увеличивается;

Б) с уменьшением давления уменьшается;

В) от давления не зависит;

Г) с увеличением давления – уменьшается.

3. Разность показаний термометров психрометра с увеличением относительной влажности…

А) не изменяется;

Б) увеличивается;

В) уменьшается;

Г) может как увеличиваться, так и уменьшаться.

4. Если водяной пар остается ненасыщенным при изменении температуры воздуха, то его абсолютная влажность будет…

А) постоянной;

Б) уменьшаться;

В) увеличиваться;

Г) принимать любые значения.

5. В сосуде, закрытом подвижным поршнем, находятся в равновесии вода и водяной пар. Объём сосуда очень медленно уменьшают, опуская поршень из положения 1 в положение 2. График зависимости давления в сосуде от объёма правильно показан на рисунке …

А) 1;

Б) 2;

В) 3;

Г) 4.

6. Относительная влажность воздуха в комнате равна 40 %. Каково соотношение концентрации n n н

А) n меньше n н в 2,5 раза;

Б) n больше n н в 2,5 раза;

В) n меньше n н на 40 %

Г) n больше n н на 40 %

7. Относительная влажность воздуха в цилиндре под поршнем равна 50 %. Воздух изотермически сжали, уменьшив его объем в 3 раза. Относительная влажность воздуха стала

А) 150 % Б) 100 %

В) 50 % Г) 25 %

    Как расположены молекулы в твердых телах и как они движутся?

А) молекулы расположены на расстояниях, меньших размеров самих молекул и перемещаются свободно друг относительно друга;

Б) молекулы расположены на больших расстояниях (по сравнению с размерами молекул) друг относительно друга и движутся беспорядочно;

В) молекулы расположены в строгом порядке и колеблются около определенных положений.

9. Какие из приведенных свойств принадлежат жидкостям? (Выберите несколько вариантов)

А) имеют определенный объем;

Б) занимают объём всего сосуда;

В) принимают форму сосуда;

Г) мало сжимаются;

Д) легко поддаются сжатию.

10. Изменится ли объем газа, если его перекачать из сосуда вместимостью 1 л в сосуд вместимостью 2 л?

А) увеличится в 2 раза;

Б) уменьшится в 2 раза;

В) не изменится.

11. Молекулы расположены на больших расстояниях друг относительно друга (по сравнению с размерами молекул), слабо взаимодействуют между собой, движутся хаотически. Какое это тело?

А) газ;

Б) твердое тело;

В) жидкость;

Г) такого тела нет.

12. В каком состоянии может находится сталь: в твердом, жидком или газообразном?

В) только в газообразном состоянии;

Г) во всех трех состояниях.

13. Ниже указано поведение молекул в твердых, жидких и газообразных телах. Что является общим для жидкостей и газов?

А) то, что молекулы расположены на расстояниях, меньших размеров самих молекул, и движутся свободно друг относительно друга;

Б) то, что молекулы расположены на больших расстояниях друг относительно друга (по сравнению с размерами молекул) и движутся беспорядочно;

В) то, что молекулы движутся беспорядочно друг относительно друга;

Г) то, что молекулы расположены в строгом порядке и колеблются около определённых положений.

14. Какие из указанных свойств принадлежат твердым телам? (Выберите несколько вариантов)

А) имеют определённый объем;

Б) занимают объем всего сосуда;

В) принимают форму сосуда;

Г) мало сжимаются;

Д) легко сжимаются.

Вариант –3

1. В бутылке находится вода объёмом 0,5 л. Её переливают в колбу вместимостью 1 л. Изменится ли объем воды?

А) увеличится;

Б) уменьшится;

В) не изменится.

2. Молекулы расположены так, что расстояние между ними меньше размеров самих молекул. Они сильно притягиваются друг к другу и перемещаются с места на место. Какое это тело?

А) газ;

Б) жидкость;

В) твердое тело.

3. В каком состоянии может находиться кислород: в твердом, жидком или газообразном?

А) только в твердом состоянии;

Б) только в жидком состоянии;

В) только в газообразном состоянии;

Г) во всех трех состояниях.

4. Что такое динамическое равновесие системы “пар-жидкость”?

А) число вылетевших молекул из жидкости при испарении больше числа молекул, вернувшихся в жидкость при конденсации;

Б) число вылетевших молекул при испарении меньше числа молекул, вернувшихся в жидкость при конденсации;

В) число молекул, покидающих жидкость за некоторое время, равно числу молекул, возвратившихся из пара в жидкость за то же время.

5. Взаимодействие между молекулами носит характер:

А) притяжения;

Б) отталкивания;

В) притяжения на малых расстояниях, отталкивания — на больших;

Г) отталкивания на малых расстояниях, притяжения – на больших.

6. При испарении от поверхности жидкости отрываются молекулы, обладающие…

А) наибольшей кинетической энергией;

Б) наименьшей кинетической энергией;

В) наибольшей потенциальной энергией;

Г) наименьшей потенциальной энергией.

7. При динамическом равновесии между жидкостью и ее насыщенным паром …

А) конденсация и испарение не происходят;

Б) конденсация происходит быстрее испарения;

В) конденсация происходит медленнее испарения;

Г) конденсация происходит с такой же скоростью, что и испарение.

8. При одной и той же температуре насыщенный пар в закрытом сосуде отличается от ненасыщенного пара

А) давлением;

Б) строением молекул;

В) средней энергией хаотичного движения молекул;

Г) отсутствием примеси посторонних газов.

9. Плотность насыщенного пара…

А) с увеличением объема – увеличивается;

Б) с уменьшением объема – уменьшается;

В) от объема не зависит;

Г) с увеличением объема – уменьшается.

10. Число молекул, покидающих поверхность жидкости, становится равным числу молекул, возвращающихся в нее, при относительной влажности…

А) 100 % Б) 50 %

В) 0 % Г) 25 %

11. В сосуде, содержащем только пар и воду, поршень двигают так, что давление остается постоянным. Температура при этом

А)не изменяется;

Б) увеличивается;

В) уменьшается;

Г) может, как уменьшаться, так и увеличиваться.

12. Относительная влажность воздуха в комнате равна 20 %. Каково соотношение концентрации n молекул воды в воздухе комнаты и концентрации n н молекул воды в насыщенном водяном паре при той же температуре?

А) n больше n н в 5 раз;

Б) n меньше n н в 5 раз;

В) n меньше n н на 20 %;

Г) n больше n н на 20 %.

13. Относительная влажность воздуха в цилиндре под поршнем равна 60 %. Воздух изотермически сжали, уменьшив его объём в два раза. Относительная влажность воздуха стала

А) 120 % Б) 100 %

В) 60 % Г) 30 %

14. В сосуде под поршнем находится ненасыщенный пар. Его можно перевести в насыщенный…

А) повышая температуру;

Б) добавляя в сосуд другой газ;

В) увеличивая объем пара;

Г) уменьшая объем пара.

Ответы:

№ вопроса Варианта

Вариант 1

А,Г,Д

А,В,Д

Вариант 2

А,В,Г

А,Г

Вариант 3


Еще по теме статьи:

Страница не найдена | Муниципальное общеобразовательное учреждение «Тоншаевская средняя школа»

Приказ МОУ Тоншаевская СОШ «Об утверждении регламента работы учреждения в условиях распространения новой коронавирусной инфекции (COVID — 19) с 9 ноября 2020 г.

Приказ МОУ Тоншаевская СОШ от 03.11.2020 г.

Приказ Управления образования, спорта и молодежной политики администрации Тоншаевского муниципального района об обучении с 5 ноября 

Приказ Министерства образования, науки и молодежной политики Нижегородской области об организации образовательной деятельности с 5.11.2020

Указ Губернатора от 28.10.2020 г.


Телефонные “горячие линии” по вопросам организации дистанционного обучения школьников: 

Управление образования, спорта и молодежной политики администрации Тоншаевского муниципального района: 8(831)-512-16-91, Смирнова Лидия Николаевна с 8.00 до 12.00 с понедельника по пятницу;  2px» original-text-shadow=»none»>8(831)-512-17-58, Чернышова Светлана Александровна с 12.00 до 16.00 с понедельника по пятницу.

МОУ Тоншаевская СОШ: 8(831)-512-16-02 с 8.00 до 12.00 с понедельника по пятницу.

Министерство образования, науки и молодёжной политики Нижегородской области: 8(831)-433-54-51 с 15.00 до 17.00 с понедельника по пятницу.

2px» original-text-shadow=»none»>ГБОУ ДПО «Нижегородский институт развития образования»: 8(831)-417-76-54 с 10.00 до 15.00 с понедельника по пятницу.

Горячая телефонная линия по вопросам классного руководства

Горячая телефонная линия по вопросам классного руководства Управления образования, спорта и молодёжной политики администрации Тоншаевского района

Горячая телефонная линия по вопросам классного руководства Министерства образования, науки и молодежной политики Нижегородской области для педагогических работников

Контрольно-измерительные материалы по физике. 7 класс

КИМ № 1 по теме «Физика и физические методы изучения природы»

Вариант 1

Что из перечисленного относится к веществу?

А. Стакан Б. Стекло В. Стена Г. Стул

Какова цена наименьшего деления

мензурки, изображенной на рис. 1?

А. 1000 см3

Б. 100 см3

В. 500 см3

Г. 10 см3

3. Определите объем воды, находящейся в мензурке (рис. 1, слева).

A.1000 см3 Б. 490 см3 B.500 см3

Г. 480 см3

4.В ту же мензурку с водой опущено тело (рис. 1,справа). Каков объем воды и тела вместе?

А. 300 см3Б. 1000 см3В. 750 см3Г. 800 см3

5.Каков объем тела?

А. 310 см3Б. 400 см3В. 300 см3Г. 800 см3

6.Что из перечисленного соответствует световому явлению?

A. Движение Луны вокруг Земли
Б. Нагревание воды в кастрюле

B.Получение на экране изображения предмета с помощью линзы
Г. Свечение электроламп

7.Установить соответствие между измерительными приборами и единицами измерения физических величин.

A. Часы 1. см
Б. Мензурка 2. сек

B. Мерная лента 3. см3

8.Установите соответствие между физическими явлениями и их харак­терными признаками.

A. Механическое1. Изменение температуры

Б. Тепловое 2. Движение заряженных частиц

B. Электрическое3. Изменение положения тел в пространстве

9. Объем капли воды равен 8 мм3. Выразите этот объем в кубических сантиметрах;

в кубических метрах.

Используя ученическую линейку, определите толщину листа в учебнике. Результаты измерений и вычислений записать в тетради.

Результат выразить в мм, см, м.

КИМ № 1 по теме «Физика и физические методы изучения природы»

Вариант 2

1.Что из перечисленного относится к телу?

А . СтеклоБ. МедьВ. АлюминийГ. Вилка

Какова цена наименьшего деления мензурки, изображенной на рисунке 2?

А. 10 см3Б. 1 см3В. 2 см3Г. 100 см3

Определите объем воды, находящейся в мензурке (рис. 2, слева).

А. 100 см3Б. 70 см3В. 10 см3Г. 80 см3

В ту же мензурку с водой опущена гайка (рис. 2, справа). Каков объем воды и гайки вместе?

А. 50 см3 Б. 60 см3 В. 80 см3 Г. 70 см3

5. Каков объем гайки?

А. 10 см3 Б. 100 см3 В. 80 см3 Г. 55 см3

6. Что из перечисленного соответствует механическому явлению?

A. Движение Луны вокруг Земли
Б. Нагревание воды в кастрюле

B.Получение на экране изображения предмета с помощью линзы
Г. Свечение электроламп

7.Установить соответствие между измерительными приборами и единицами измерения физических величин.

A. Секундомер 1. мм
Б. Ученическая линейка 2. г

B. Весы 3. сек

8.Установите соответствие между физическими явлениями и их харак­терными признаками.

A. Звуковые 1. Изменение положения тел в пространстве
Б. Световые 2. Колебание тела

B.Механические 3. Образование на освещенном экране тени от предмета

9. Площадь тетрадного листа равна 750 см2. Выразите эту площадь в квадратных дециметрах; в квадратных метрах.

10.Используя ученическую линейку и карандаш, определите толщину
нитки. Результаты измерений и вычислений записать в тетради. Результат выразить в мм, см, м.

КИМ № 2 по теме «Строение вещества»

Вариант 1

1. Может ли капля растительного масла беспредельно растекаться по поверхности воды?

A. Может. Ей ничто не препятствует.

Б. Нет. Будет растекаться до тех пор, пока толщина слоя не окажется равной размерам наименьшей частицы масла.

B.Капля масла растекаться не будет, на поверхности воды капля примет форму
пирамидки.

Г. Среди ответов А-В нет правильного.

2.В каких веществах (твердых, «жидких или газообразных) происходит диффузия?

A. Диффузия происходит только в газах.

Б. Диффузия происходит только в жидкостях.

B.Диффузия происходит только в твердых телах.

Г. Диффузия происходит в твердых, жидких и газообразных телах.

3.Для какой цели нагревают стальные детали и хромовый порошок?

A. Увеличивается расстояние между молекулами стали, и между ними проникают частицы хрома.

Б. Увеличивается скорость движения молекул обоих веществ, и быстрее проте­кает диффузия.

B.Образуется сплав хрома и стали.

Г. Среди ответов А-В нет правильного.

4.Какие из приведенных свойств принадлежат газам?

A. Занимают весь предоставленный им объем.
Б. Трудно сжимаются.

B.Имеют кристаллическое строение.
Г. Имеют определенную форму.

5.Молекулы плотно упакованы, сильно притягиваются друг к другу, и каждая молекула колеблется около определенного положения. Какое это тело?
А. Газ. Б. Жидкость. В. Твердое тело.Г. Таких тел нет.

6. Между молекулами в веществе…

A. Существует взаимное притяжение и отталкивание.
Б. Не существует ни притяжения, ни отталкивания.

B.Существует только притяжение.
Г. Существует только отталкивание.

7.Установите соответствие между агрегатным стоянием вещества и движением молекул.

A. Твердое 1. Движутся беспорядочно, хаотически

Б. Жидкое 2. Совершают колебания около положения равновесия

B.Газообразное 3. Колеблются между молекулами, перескакивая из одной точки

пространства в другую

Изменится ли объем газа, если его перекачать из баллона вместимо­стью 20 л в баллон вместимостью 40 л? Если изменится, то каким образом и на сколько?

Используя стакан с горячей водой, стакан с холодной водой и кристал­лический арганец, исследуйте, как зависит скорость диффузии от температу­ры жидкости. Опишите последовательность проведения эксперимента. Опиши­те наблюдаемое явление. Сделайте вывод. Укажите практическое применение полученной закономерности.

10.Какую площадь поверхности займет разлившаяся по ней нефть объемом 1 м3 при толщине слоя в 1/40000 мм?

КИМ № 2 по теме «Строение вещества»

Вариант 2

1. Все молекулы одного и того же вещества…

A. Не отличаются друг от друга
Б. Отличаются друг от друга

B.Молекулы имеют одинаковые размеры
Г. Среди ответов А-В нет правильного

2.Что такое диффузия?

A. Явление проникновения молекул одного вещества между молекулами другого
Б. Явление, при котором вещества смешиваются друг с другом

B.Явление, при котором вещества сами собой смешиваются друг с другом
Г. Среди ответов А-В нет правильного

3.Для того чтобы свежие огурцы быстрее засолились, их заливают горячим рассолом. Почему засолка огурцов в горячем рассоле протекает быстрее?

A. Быстро растворяется соль

Б. Расстояние между молекулами клетчатки огурцов становится больше, и сам процесс протекает быстрее

B.Скорость движения молекул увеличивается, и диффузия протекает быстрее

Г. Среди ответов А-В нет правильного

4.Какие из указанных свойств принадлежат твердым телам?

A. Имеют определенный объем Б. Занимают объем всего сосуда

B.Принимают форму сосуда Г. Легко сжимаются

5.Молекулы расположены на больших расстояниях друг относительно друга (по сравнению с размерами молекул), слабо взаимодействуют между собой, движутся хаотически. Какое это тело?

А. Газ Б. Твердое телоВ. Жидкость Г. Такого тела нет

6.Молекулы притягиваются друг к другу. Но почему между ними существуют промежутки, и они не «слипаются» между собой? Это происходит потому, что они …

A. Движутся

Б. Очень слабо притягиваются друг к другу

B.При большом сближении отталкиваются друг от друга
Г. Среди ответов А-В нет правильного

7.Установите соответствие между агрегатным стоянием вещества и расстоянием между молекулами.

A. Твердое1. Молекулы расположены на расстояниях, меньших размеров самих молекул.
Б. Жидкое2. Молекулы расположены на больших расстояниях (по сравнению с размерами молекул) друг от друга.

B.Газообразное 3. Молекулы расположены в строгом порядке.

В бутылке находится вода объемом 0,5л. Ее переливают в колбу вме­стимостью 1 л. Изменится ли объем воды? Если изменится, то каким образом и на сколько?

Используя стакан с водой, пробирку, заткнутую пробкой со стеклянной трубкой, исследуйте, как зависит давление воздуха в пробирке от его темпера­туры. Опишите план проведения эксперимента. Сделайте схематический рису­нок экспериментальной установки. Опишите наблюдаемые явления. Сделайте вывод.

Капля масла объемом 3 mmj растеклась по поверхности воды, образовав пятно площадью 2000 см2. Чему равен диаметр молекулы масла?

КИМ № 3 по теме «Механические явления»

Вариант 1

1.Изменение положения тела относительно другого тела с течением времени называют

A. Пройденным путемБ. Траекторией

B.Механическим движениемГ. Перемещением

2.Какое движение называют равномерным?

A. Движение, при котором тело в любые равные промежутки времени проходит
равные пути

Б. Движение, при котором тело в равные промежутки времени проходит равные пути

B.Движение, при котором тело движется так, что его траектория — прямая линия

Г. Движение, при котором тело за любые равные промежутки времени совер­шает различные перемещения

3.Как называют линию, которую описывает тело при своем движении?

A. Прямой линиейБ. Пройденным путем

B.Траекторией Г. Окружностью

4.Скорость тела при равномерном движении показывает

A. Время, затраченное телом на прохождение единицы пути
Б. Какой путь проходит тело в единицу времени

B.Какой путь проходит тело за время своего движения

Г. Сколько времени затрачивает тело на прохождения всего пути

5.Из стали, чугуна, латуни и алюминия изготовлены шарики одинаковой массы. Какой из них имеет меньшие размеры?

А. Чугунный Б. ЛатунныйВ. Стальной Г. Алюминиевый.

6.Чтобы определить плотность вещества, надо …
А. Его массу разделить на объем

Б. Его объем разделить на массу

В. Его массу умножить на объем

Г. Среди ответов А-В нет правильного

7. Установите соответствие между физическими величинами и единицами их измерения.

A. Путь 1. кг
Б. Скорость 2. м

B.Масса 3. м/с

4. с

8.За какое время велосипедист проедет 250 м, двигаясь со скоростью 5 м/с?

9.Используя измерительную линейку, секундомер, стеклянную трубку
длиной 200-250 мм и внутренним диаметром 7-8 мм, заполненную водой с пузырьком воздуха, определите скорость движения пузырька воздуха в стеклянной трубке, заполненной водой (трубка расположена вертикально). Сделайте
схематический рисунок. Результаты измерений запишите в таблицу, запишите
расчетные формулы, по результатам измерений вычислите скорость пузырька
воздуха.

10.Машина рассчитана на перевозку груза массой 3 т. Сколько листов
железа можно погрузить на нее, если длина каждого листа
-2м, ширина — 80см, а толщина — 2 мм? Плотность железа — 7800 кг/м3.

КИМ № 3 по теме «Механические явления»

Вариант 2

1.Что называют механическим движением?

A. Длину траектории, по которой движется тело

Б. Изменение положения тела относительно других тел

B.Линию, по которой движется тело

Г. Расстояние между двумя точками в пространстве

2.Движение, при котором тело за любые равные промежутки времени совершает одинаковые перемещения, называют…

A. Механическим движением Б. Равномерным движением

B.Неравномерным движением Г. Движением тела по окружности

3. Линию, которую описывает тело при своем движении в течение некоторого промежутка времени, называют

A. Пройденным путем Б. Траекторией

B.Механическим движением Г. Перемещением

4. Скоростью равномерного движения называют величину, численно рав­ную…

A. Времени прохождения телом единицы пути
Б. Пути, пройденному телом за время движения

B.Пути, проходимому телом в единицу времени

Г. Времени, затрачиваемому телом на прохождения всего пути

5.Стальной, алюминиевый, латунный и чугунный шарики имеют одинаковые объемы. Какой из них имеет большую массу?

А. Алюминиевый Б. Стальной В. Латунный Г. Чугунный

6.Плотность льда 900 кг/м3. Это означает, что …

A.В объеме 1 м3 содержится лед массой 900 кг
Б. Лед массой 1 кг занимает объем 900 м3

B. Лед массой 900 кг занимает объем 900 м3
Г. Среди ответов А-В нет правильного.

7.Установите соответствие между физическими величинами и единицами их измерения.

А. Сила 1. с

Б. Плотность 2. кг

В. Время 3. кг/м3

4. Н

8. За какое время самолет, движущийся со скоростью 200 м/с, пройдет путь, равный

3000 м?

9.Используя штатив с муфтой и лапкой, желоб, металлический шарик, секундомер, мерную ленту, определите среднюю скорость скатывания шарика по желобу. Сделайте схематический рисунок. Результаты измерений запишите в таблицу, запишите расчетные формулы, по результатам измерений вычислите среднюю скорость скатывания пузырька.

10.Объем легких у человека 3000 см3. За одну минуту в его легкие поступает 77,4 г воздуха. Сколько вдохов в минуту делает человек? Плотность воздуха 1,29 кг/м3

КИМ № 4 по теме «Механические явления»

( Силы в природе)

Вариант 1

1. Явление сохранения скорости тела при отсутствии действия на него других тел называют…

A. Механическим движением Б. Инерцией

B.Движением тела Г. Диффузией

2.Изменение скорости движения тела происходит…

A. Пока действует на него другое тело
Б. Без действия на него другого тела

B.После действия на него другого тела

Г. Среди ответов А-В нет правильного *

3.На гирю, лежащую на столе, действуют … . Эти силы …. по модулю и имеют… направления.

A. Сила тяжести и вес тела … не равны … одинаковые …

Б. Сила упругости и вес тела … равны … противоположные …

B.Силы тяжести и сила упругости … равны … противоположные …
Г. Среди ответовА-В нет правильного.

4.Силу, которую производит на тело такое же действие, как несколько одновременно действующих сил, называют ….., а находят ее — …

A. Суммой этих сил… равнодействующей

Б. Сложением этих сил… равнодействующей

B.Равнодействующей этих сил… сложением этих сил
Г. Составляющей… суммой этих сил

5.Какая сила удерживает тело на поверхности Земли?

A. Сила упругостиБ. Вес тела

B. Сила тяжестиГ. Среди ответов А-В нет правильного

6.Сани скатываются с горы под действием силы а скатившись, останавливаются за счет силы …

A. трения … тяжестиБ. упругости … трения

B.трения … упругостиГ. тяжести … трения

7.Установите соответствие между силой и причинами ее проявления.

A. Сила тяжести 1. Шероховатости соприкасающихся поверхностей
Б. Сила упругости 2. Притяжение тел к поверхности земли

B. Сила трения 3. Изменение характера взаимодействия между молекулами при деформации тела

Пружина жесткостью 40 Н/м под действием некоторой силы удлини­лась на 5 см. Чему равна величина силы упругости пружины при ее удлинении?

Используя брусок и динамометр, измерьте вес бруска и силу трения бруска по поверхности стола. Запишите, чему равны цена деления и предел из­мерения шкалы динамометра, вес бруска и сила трения. Вычислите отношение силы трения к весу бруска.

10.Чему равна масса тела, если его вес равен 5 Н? Ускорение свободного
падения считать равным 10 Н/кг.

КИМ № 4 по теме «Механические явления»

( Силы в природе)

Вариант 2

1. Как называется явление сохранения скорости тела при отсутствии действия на него других тел?

A. Механическим движениемБ. Инерцией

B. Движением телаГ. Диффузией

2.Известно, что тело может изменить свою скорость только под действием других тел. Почему же человек при ходьбе сам может изменять свою скорость?

A.Человек — разумное существо, и он изменяет свою скорость по желанию
Б. Человек взаимодействует с землей

B.Человек при ходьбе может шагать быстрее или медленнее, не взаимодействуя с другими телами

Г. Среди ответов А-В нет правильного

3.Металлический шарик висит на нити. В точке подвеса действуют…..

Эти силы …. по модулю и имеют …. направление.

A. Сила тяжести и вес тела… не равны… одинаковые…

Б. Сила упругости и вес тела… равны… противоположные…

B.Силы тяжести и сила упругости… равны… противоположные…
Г. Среди ответов А-В нет правильного.

4. Равнодействующая двух сил, направленных по одной прямой в одну сторону, равна … этих сил и направлена …

A. Сумме… в ту же сторону.

Б. Разности… в сторону большей силы.

B. Сумме… в сторону меньшей силы.
Г. Разности… в сторону меньшей силы.

5.Тело, выпущенное из рук, падает на землю. Какая сила вызывает падение тел?

A .Сила тяжестиБ. Сила упругости

B.Вес тела Г. Среди ответов А-В нет правильного.

6.Два биллиардных шара, столкнувшись, отталкиваются друг от друга за счет силы…., а затем останавливаются за счет силы….

A. трения… тяжестиБ. упругости… трения

B. трения… упругостиГ. упругости… тяжести

7. Установите соответствие между силой и формулой для ее вычисления.

A. Сила тяжести1.F=k* ∆1
Б. Сила упругости 2.F=μ* Р

B.Сила трения 3. F = m*g

Пружина жесткостью 100 Н/м под действием некоторой силы удлини­лась на 2 см. Чему равна величина силы упругости пружины при ее удлинении?

Используя деревянный брусок, целлофановую обложку от тетради и динамометр, измерьте вес бруска и силу трения бруска по целлофановой об­ложке тетради. Запишите, чему равны цена деления и предел измерения шкалы динамометра, вес бруска и сила трения. Вычислите отношение силы трения к весу бруска.

Чему равна масса тела, если его вес равен 15 Н? Ускорение свободного падения считать равным 10 Н/кг/

КИМ № 5 по теме «Механические явления»

( Давление твердых тел, жидкостей и газов)

Вариант 1

1.От чего зависит результат действия силы на тело?

A.От модуля силы и от площади поверхности, перпендикулярно к которой она
действует.

Б. От площади поверхности, перпендикулярно к которой действует сила.

B.От модуля силы и не зависит от площади поверхности, на которую она действует.

Г. Не зависит ни от площади поверхности, ни от модуля силы.

2.У комбайнов, сеялок и других сельскохозяйственных машин колеса де-
лают с широкими ободами для того, чтобы … давление, так как чем … площадь
опоры, тем … давление
.

A. Уменьшить… меньше… меньше
Б. Уменьшить… больше… меньше

B.Увеличить… меньше… больше
Г. Увеличить… больше… больше

3.Зависит ли давление жидкости на дно сосуда от площади дна?
А. А. Не зависит

Б. Тем больше, чем больше площадь

В.Тем меньше, чем меньше площадь

Г. Среди ответов А-В нет правильного ответа

4. Что можно сказать о давлении газа в сосуде по сравнению с атмосферным давлением (рис. 6)?

A.Оно меньше атмосферного
Б. Оно равно атмосферному

B.Оно больше атмосферного
Г. Однозначный ответ нельзя дать

5.Чем сильнее сжат газ, тем … его плотность и тем-… давление, которое он производит.

A. Больше… меньше Б. Больше… больше

B.Меньше… больше Г. Меньше… меньше

6.Выразите давление 760 мм рт. ст. в Паскалях.

А. ≈104000 Па; Б. ≈ 118000 Па; В. ≈ 28 000 ПаГ.≈101 000 Па;

7.Установите соответствие между агрегатным состоянием вещества и давлением, оказываемым телом.

A. Твердые тела 1. По всем направлениям

Б. Жидкости 2. В направлении действия силы тяжести

B.Газы 3. На дно и стенки сосуда


У подножья горы барометр показывает давление 760 мм рт. ст., а на вершине горы

722 мм рт. ст.. Какова примерно высота горы?

Используя барометр-анероид, масштабную линейку, вычислите силу давления, оказываемую атмосферой на поверхность тетрадного листа. Резуль­таты измерений запишите в таблицу. Запишите расчетные формулы для вычис­ления площади тетрадного листа и силы давления. Укажите возможные спосо­бы увеличения и уменьшения силы давления на поверхность тетрадного листа при постоянной его площади поверхности.

10.Гранитная плита лежит на земле, опираясь на грань, имеющую длину 1,2 м и ширину 0,8 м. Определите давление плиты на землю, если ее масса 480 кг.

КИМ № 5 по теме «Механические явления»

( Давление твердых тел, жидкостей и газов)

Вариант 2

1. Давлением называют величину, равную

A.Силе, действующей на единицу площади опоры

Б. Отношению силы, действующей перпендикулярно к поверхности, к площади этой поверхности

B.Отношению силы, действующей на поверхность, к площади этой поверхности

Г. Отношение площади опоры к действующей силе

2. Режущие инструменты затачивают для того, чтобы … давление, так как,
чем … площадь опоры, там … давление.

A. Уменьшить… меньше… меньшеБ. Уменьшить… больше… меньше

B.Увеличить… меньше… больше Г. Увеличить… больше… больше

3.Зависит ли давление жидкости на дно и стенки сосуда от плотности жидкости?

A. Не зависит

Б. Давление жидкости прямо пропорционально плотности жидкости

B.Давление жидкости обратно пропорционально плотности жидкости
Г. Среди ответов А-В нет правильного ответа

4.Что можно сказать о давлении газа в сосуде по
сравнению с атмосферным давлением (рис. 7)?

A. Оно меньше атмосферного давления
Б. Оно равно атмосферному давлению

B.Оно больше атмосферного давления
Г. Однозначный ответ нельзя дать

5.Чем дальше слой воздуха от поверхности Земли, тем … он сжат, тем … его плотность и, следовательно, тем … давление он производит.

A. Сильнее… меньше… большее
Б. Слабее… больше… меньшее

B.Слабее… меньше… меньшее
Г. Сильнее… больше… меньшее

6.Атмосферное давление равно 780 мм рт. ст. Выразите его в паскалях.р,к

А. ≈ 105 ООО ПаБ. ≈ 98600 Па В.≈ 20000 Па Г. ≈ 104 ООО Па

7. Установите соответствие между измеряемой величиной и прибором для ее измерения.

A. Атмосферное давление 1. Высотомер
Б. Давление жидкости или газа на стенки сосуда 2. Секундомер

B.Высоту полета самолета 3. Манометр

4. Барометр

8.У подножья горы барометр показывает давление 760 мм рт. ст., а на вершине горы

700 мм рт. ст.. Какова примерно высота горы?

9.Измерьте наибольшее давление бруска на поверхность стола используя линейку с миллиметровыми делениями и динамометр. Сделайте схематический рисунок, укажите, в каком случае давление будет наибольшим. Результаты из­мерений запишите в таблицу. Запишите формулу для вычисления давления те­ла на поверхность.

10.Гусеничный трактор весом 54 кН в среднем производит давление 40000 Па. Определите опорную площадь гусениц.

КИМ № 6 по теме «Механические явления»

( Выталкивающая сила. Плавание тел)

Вариант 1

1. К одинаковым пружинам подвешены шарики равной массы,

но разного объема. Снизу к шарикам подносят сосуд с водой

и поднимают его до такого уровня, когда шарики полностью

погрузятся в воду(рис. 9). Какая пружина сократится больше?
А. ПерваяБ. ВтораяВ. Третья

Г. Все пружины сократятся одинаково

2.В сосуд с керосином погружен параллелепипед на глубину,

указанную на рисунке 10. Площадь верхней и нижней грани

по 0,005 м2Что можно сказать о силах, действующих
на правую и левую, а также на переднюю и заднюю
грани параллелепипеда?

A.Силы, действующие на правую и левую грани,
по модулю равны, равны также и модули сил, действующих

на переднюю и заднюю грани.
Б. Сила, действующая на правую грань, больше, чем
на левую, а на переднюю — больше, чем на заднюю.

B.Сила, действующая на правую грань, больше,
чем на левую, а на переднюю и заднюю грани — равны.

Г. Силы, действующие на правую грань, больше, чем на левую грань, а силы, действующие на переднюю и заднюю грани, равны по модулю

Вычислите силу, действующую на верхнюю грань параллелепипеда.

А.7НБ. 8 НВ. 16 НГ. 4000 Н

С какой силой параллелепипед выталкивается из керосина?

А. 2000 НБ. 4 НВ. 800 НГ. 1000 Н

Медный цилиндр массой 1,78 кг опущен в бензин. Вычислите объем вытесненного им бензина.

А. 2 м3 Б. 1,8 м3 В. 0,02 м3 Г. 0,0002 м3

6. В какой жидкости утонет кусок парафина?

А. В водеБ. В ртутиВ. В морской воде Г. В бензине

7.Установите соответствие между условиями плавания тел и соотношением силы тяжести тела и силы Архимеда.

A. Плавает 1. Fтяж >Fa
Б. Тонет 2. Fтяж = Fa

B.Всплывает 3. Fтяж< Fa

4. Ни одно из рассмотренных условий не соответствует условиям плавания тел

8. В воду погрузили тело объемом 120 см3. Определите значение выталкивающей силы, действующей на тело.

9.Используя динамометр, сосуд с водой и тело неправильной формы, определите объем данного тела. Сделайте схематический рисунок. Запишите результаты измерений. Запишите расчетную формулу. По результатам измерений вычислите объем тела. Значение объема укажите в см3, м3.

10.Объем тела человека — 0,06 м3, а средняя плотность — 1070 кг/м3. С какой силой человек давит на дно реки, погрузившись до половины своего объема?

КИМ № 6 по теме «Механические явления»

( Выталкивающая сила. Плавание тел)

Вариант 2

1. К одинаковым по упругости пружинам подвешены тела

равной массы и равного объема (рис. 11). Какая пружина

станет самой короткой, если тела погрузить в жидкости?

А. Первая Б. Вторая В. Третья

Г. Все пружины будут иметь одинаковую длину

2.В сосуд с водой погружен параллелепипед на глу­бину,

указанную на рисунке 12. Площади его оснований по 0,5 м2.

Что можно сказать о модулях сил, действующих на правую

и левую, а также на переднюю и заднюю гра­ни параллелепипеда?

А. Силы, действующие на правую и левую грани, по мо­дулю

равны, равны по модулю и силы, действующие на переднюю

и заднюю грани

Б. Сила, действующая на правую грань, по модулю больше,

чем на левую, а на переднюю — больше, чем на заднюю

В Сила, действующая на правую грань, больше, чем на левую,

а на переднюю и заднюю грани — равны

Г. Силы, действующие на правую грань больше чем на левую

грани, а силы, действующие на переднюю и зад­нюю грани равны по модулю

Вычислите силу, действующую на верхнее основание параллелепипеда.

А.7НБ. 8 НВ. 16 НГ. 4000 Н

С какой силой параллелепипед выталкивается из воды?

А. 2000 НБ. 4 Н В.800 НГ. 1000 Н

Кусок алюминия массой 540 г опущен в керосин. Каков объем вытес­ненного керосина?

А. 2 м3Б. 1,8 м3В. 0,02 м3Г. 0,0002 м3

6.В какой жидкости не утонет лед?

А. В нефти Б. В ацетоне В. В спирте Г. В морской воде

7.Установите соответствие между формулой, описывающей физический
закон, и физической величиной, которую можно вычислить с помощью этой
формулы
.

A.ρ = pg h2. Сила Архимеда

Б. Fa = ρgVж2. Давление некоторой силы на площадку В. p = F / S 3. Давление столба жидкости

4. Давление воздушного столба

Чему равна архимедова сила, действующая на тело объемом 200 см , полностью погруженное в керосин?

Используя динамометр, сосуд с водой и тело неправильной формы, оп­ределите плотность вещества, из которого изготовлено данное тело. Сделайте схематический рисунок. Запишите результаты измерений. Запишите расчетные формулы. По результатам измерений вычислите объем тела. (Плотность воды принять равной

1000 кг/м3.)

10.Стальной якорь, имеющий вес в воздухе 3950 Н, находится на дне реки. Какую силу надо приложить к цепи якоря, чтобы поднять его?

КИМ № 7 по теме «Механические явления»

( Работа, мощность и энергия)

Вариант 1

1.Вкаком случае совершается работа?

A. Шарик катится по гладкому горизонтальному столу
Б. Кирпич лежит на земле

B.Автопогрузчик поднимает груз

Г. Человек, стоя на месте, держит на плечах груз

2.Мощность численно равна работе, совершенной ..

A. В одну секунду Б. Машиной

B.В одну минуту Г. За один час

3.Какой энергией относительно Земли обладает летящий самолет?

A. Потенциальной Б. Кинетической

B.Потенциальной и кинетической
Г. Среди ответов А-В нет правильного

4.При падении тела … энергия переходит в …

A. Потенциальная… кинетическуюБ. Кинетическая… потенциальную

B.Кинетическая… кинетическую Г. Потенциальная… потенциальную

5.На рисунке 15 изображена тележка, движущаяся при опускании груза.
В этом устройстве … энергия груза переходит в … энергию тележки.

A. Потенциальная … кинетическую Б. Кинетическая … потенциальную

B.Потенциальная … потенциальную Г. Кинетическая … кинетическую

6.Велосипедист за 5 с совершил работу 400 Дж. Вычислите мощность велосипедиста.

А. 2000 Вт.Б. 80 Вт.В. 0,0125 Вт.Г. 405 Вт

7. Установите соответствие между формулой, описывающей физический
закон, и физической величиной, которую можно вычислить с помощью этой
формулы.

A.А = F * s1. Потенциальная энергия поднятого тела

Б. N = A / t 2.Кинетическая энергия движения тела

B.Еп = mgh4. Механическая работа

4. Мощность

Человек массой 60 кг поднялся по лестнице на высоту 5 м. На сколько изменилась потенциальная энергия человека?

Используя штатив с муфтой и лапкой, рейку трибометра, деревянный брусок, динамометр и мерную ленту, вычислите КПД при подъеме тела на не­которую высоту по наклонной плоскости. Сделайте схематический рисунок. Запишите результаты измерений. Запишите расчетные формулы. По результа­там измерений вычислите полезную работу, затраченную работу, КПД наклон­ной плоскости.

10.Неподвижным блоком равномерно поднимают груз массой 72 кг на высоту 2 м, затрачивая работу 1600 Дж. Вычислите КПД блока.

КИМ № 7 по теме «Механические явления»

( Работа, мощность и энергия)

Вариант 2

1.В каком из перечисленных случаев совершается работа?

A. Тело, выпущенное из рук, падает на землю
Б. На столе стоит гиря

B.По гладкой горизонтальной поверхности стекла катится шарик
Г. На нитке подвешен груз

2.Мощность — это величина, характеризующая …

A. Время выполнения работы

Б. Количество выполненной работы

B.Быстроту выполнения работы

Г. Среди ответов А-В нет правильного

3.Энергией какого вида обладает молот при падении?

A.ПотенциальнойБ. Кинетической

B.Потенциальной и кинетическойГ. Среди ответов А-В нет правильного

4. Когда санки скатываются с горы, то … энергия переходит в …

A.Потенциальная… кинетическую. Б. Кинетическая… потенциальную.

B.Кинетическая… кинетическую.Г. Потенциальная… потенциальную.

5. Когда ветер согнул ветку дерева, то … энергия воздуха перешла в …
энергию ветки.

A.Потенциальная… кинетическуюБ. Кинетическая… потенциальную

B.Кинетическая… кинетическуюГ. Потенциальная… потенциальную

6. Стогометатель поднимает копну сена массой 120 кг на высоту 5 м за 6с. Какую мощность развивает двигатель трактора, приводящий в движение стогометатель?

А. 1000 ВтБ. 1440 ВтВ. 2,5 ВтГ. 40 Вт

7.Установите соответствие между физическими величинами и единицами
их измерения
.

A. Дж 1. Масса
Б. Вт 2. Время

B.кг3. Мощность

4. Механическая работа

Боек копра массой 250 кг поднят на высоту 5 м относительно забивае­мой им сваи. Вычислите энергию бойка относительно сваи.

Используя штатив с муфтой и лапкой, подвижный блок, набор грузов, капроновую нить с колечками на концах, динамометр и мерную ленту, вычис­лите КПД при подъеме тела на некоторую высоту при помощи подвижного блока. Сделайте схематический рисунок. Запишите результаты измерений. За­пишите расчетные формулы. По результатам измерений вычислите полезную работу, затраченную работу, КПД при подъеме тела при помощи подвижного блока

Грузоподъемник с электролебедкой поднял груз массой 200 кг на высоту 20 м, при этом электродвигатель совершил работу 48 кДж. ВычислитеКПД электролебедки.

КИМ № 8 Итоговая контрольная работа за курс физики 7 класса

Вариант 1

1.Относительно каких тел пассажир, сидящий в движущемся вагоне, находится в состоянии покоя?

A. Вагона Б. Земли

B.Колес вагона Г. Пассажиров, стоящих на перроне

2.Какое из перечисленных движений — равномерное?

A. Движение автомобиля при торможении
Б. Движение маятника в часах

B.Течение воды в равнинной реке
Г. Движение пули в стволе винтовки

За какое время велосипедист проедет 250 м, двигаясь со скоростью 5 м/с?

А. 1250 сБ. 20 сВ. 50 сГ. 30 с

На тело действует сила 9 Н. Какую силу надо приложить, чтобы равно­действующая совпадала с этой силой по направлению и была бы равна 7 Н?

A.16 Н в сторону, противоположную силе 9 Н
Б. 2 Н в ту же сторону, что и сила 9 Н

B.16 Н по направлению силы 9 Н

Г. 2 Н в сторону, противоположную силе 9 Н

5.Чем дальше слой воздуха от поверхности Земли, тем … он сжат, тем …
его плотность, и, следовательно, тем … давление он производит.

A. Сильнее… меньше… большееБ. Слабее… больше… меньшее

B.Слабее… меньше… меньшееГ. Сильнее… больше… меньшее

6.Велосипедист, двигаясь с постоянной скоростью, за 5 с совершил работу 400 Дж. Вычислите мощность, развиваемую велосипедистом во время движения.

А. 80 Вт;Б. 2000 Вт;В. 50 Вт; Г. 8 Вт.

7.Установить соответствие между измерительными приборами и единицами измерения физических величин.

А. Барометр 1. см

Б. Часы 2. см3

В. Ученическая линейка 3. сек

4. Па

8. Установите соответствие между формулой и физической величиной, которую можно вычислить с помощью этой формулы.
А.А = F * s1. Сила Архимеда

Б.Fa = ρgVж 2.Мощность

В. . N = A / t 3. Механическая работа

4. Давление воздушного столба

9. Объем легких у спортсменов в 2tраза больше, чем у людей, не занимающихся спортом. Вычислите массу воздуха, вдыхаемого спортсменом при одном вдохе, если у людей, не занимающихся спортом, объем легких — 3000см . Плотность воздуха — 1,29 кг/м3.

10. Перемещая длинное плечо рычага, совершают работу 240 Дж. Груз какой массы поднимают этим рычагом на высоту 0,2 м, если он прикреплен к короткому плечу?

КИМ № 8 Итоговая контрольная работа за курс физики 7 класса

Вариант 2

1. Какие тела или части тел находятся в покое относительно Земли?

A. Нижние части гусениц движущегося трактора
Б. Верхние части гусениц движущегося трактора

B.Солнце

Г. Человек, идущий по дороге

2.Какие движения неравномерные?

A. Движение секундной стрелки часов

Б. Движение шарика, выпущенного из рук

B. Движение эскалатора метро

Г. Течение воды в равнинной реке

3. За какое время самолет, движущийся со скоростью 200 м/с, пройдет путь, равный

3000 м?

А. 150 сБ. 15 сВ. 6000 сГ. 60 с.

4. Грузовой состав из 80 вагонов тянут два тепловоза силой 250 и 110 кН.
Чему равна равнодействующая сил, действующих на состав со стороны тепловозов?

А. 1400 кНБ. 500 кНВ. 360 кНГ. 140 кН.

5. Чем сильнее сжат газ, тем … его плотность и тем … давление, которое он производит.

A. Больше… меньшеБ. Больше… больше

B.Меньше… большеГ. Меньше… меньше

6.Человек, поднимаясь по лестнице в течение 40 с, совершил работу 2000 Дж. Какую мощность развивал человек?

А. 50 ВтБ. 80 ВтВ. 500 Вт Г. 80 кВт.

7.Установить соответствие между измерительными приборами и единицами измерения физических величин.

A. Динамометр 1. сек
Б. Мензурка 2. см

B. Мерная лента 3. Н

4. см3

8.Установите соответствие между формулой, описывающей физический закон, и физической величиной, которую можно вычислить с помощью этой формулы.

А. ρ = m / V 1. Потенциальная энергия поднятого тела

Б. p = F / S 2. Плотность

В. En = mgh 3. Давление некоторой силы на площадку

4. Кинетическая энергия движения тела

9. Для нормальной жизнедеятельности человека необходимо 0,65 м3 ки­слорода в сутки. Вычислите массу кислорода. Плот­ность кислорода — 1,43 кг/м3.

10. Пользуясь шестом как рычагом, человек поднял груз на высоту 0,2 м, совершив работу 280 Дж. (рис.18) Длинное плечо рычага в 5 раз больше короткого. С какой силой человек действовал на ры­чаг?

Адрес публикации: https://www.prodlenka.org/metodicheskie-razrabotki/397537-kontrolno-izmeritelnye-materialy-po-fizike-7-

Тестовая контрольная работа по физике «Первоначальные сведения о строении вещества»

Контрольная работа №1

Первоначальные сведения о строении вещества

 

Демонстрационный вариант

 

№1 Мельчайшая частица данного вещества — …

1) ион

2) электрон

3) атом

4) молекула

 

№2 Между молекулами любого вещества действуют

1) Только силы отталкивания

2) Только силы притяжения

3) Силы притяжения и отталкивания

4) Не действуют никакие силы

 

№3 Какое из приведенных ниже высказываний относится к жидкому состоянию вещества?

1) Имеет собственную форму и объем

2) Имеет собственный объем, но не имеет собственной формы

3) Не имеет ни собственного объема, ни собственной формы

4) Имеет собственную форму, но не имеет собственного объема

 

№4 Какое из утверждений верно? При переходе вещества из жидкого состояния в твёрдое.

А.Уменьшается среднее расстояние между его молекулами;

Б.Молекулы начинают сильнее притягиваться друг к другу;

В.Образуется кристаллическая решётка.

1)Только А;

2) Только Б;

3)Только В;

4)А, Б и В.

 

№5 Объем газа, если его перекачать из баллона вместимостью 20 л в баллон вместимостью 40 л

1) Не изменится

2) Изменится на 20 л

3) Увеличится в 2 раза

4) Уменьшится в 2 раза.

 

№6 Установите соответствие между физическими понятиями и их примерами.

К каждой позиции первого столбца подберите соответствующую позицию второго и запишите в таблицу выбранные цифры под соответствующими буквами.

 

ФИЗИЧЕСКИЕ ПОНЯТИЯ

ПРИМЕРЫ

А) Физическое явление

1) Яблоко

Б) Физическое тело

2) Медь

В) Вещество

3) Молния

 

4) Скорость

 

5) Секунда

 

№7 Установите соответствие между физическими явлениями и их примерами.

К каждой позиции первого столбца подберите соответствующую позицию второго и запишите в таблицу выбранные цифры под соответствующими буквами.

ФИЗИЧЕСКИЕ ЯВЛЕНИЯ

ПРИМЕРЫ

А) Механическое явление

1) Слышны раскаты грома

Б) Звуковое явление

2) Полет птицы

В) Тепловое явление

3) Свечение электролампочки

 

4) Магнит притягивает к себе железные предметы

 

5) Плавление стали

№8 Почему соленая сельдь, после того как её оставили на некоторое время в воде, делается менее соленой?

№9 Определить цену деления барометра, изображенного на рисунке в мм рт. ст.

 

№10 Определите объем тела, погруженного в жидкость.

 

 

Критерии оценки.

Каждое задание 1-5 имеет один правильный ответ и оценивается в 1 балл.

Ответы на задания 6-7 представляют собой последовательность цифр и оцениваются в 2 балла, если все цифры верны и в 1 балл, если допущена 1 ошибка. Если ошибок 2 или больше, то задание оценивается в 0 баллов.

Задание 8 требует ответа на вопрос с объяснением, оценивается от 0 до 2 баллов.

Задания 9 – 10 в случае правильного решения оцениваются в 2 балла.

Максимальный балл за работу – 15 баллов.

Оценка «5» ставится за 13-15 баллов.

Оценка «4» ставится за 10-12 баллов

Оценка «3» ставится за 7-9 баллов

Оценка «2» ставится за результат менее 7 баллов.

Мультимедийная презентация по теме 2. 4 «Кипение. Критическое состояние вещества»

ГАПОУ НСО «Барабинский медицинский колледж»

Тема:

«Кипение. Критическое состояние вещества»

Преподаватель: Вашурина Т. В. Барабинск, 2017

Цели учебного занятия:

  • сформировать представления о роли и месте физики в современной научной картине мира; способствовать формированию умения владеть основополагающими физическими понятиями, уверенно пользоваться физической терминологией и символикой.
  • развивать коммуникативные способности; формировать умение работать в коллективе и команде.
  • способствовать формированию умения решать физические задачи.

Фронтальный опрос Как расположены молекулы газов, жидкостей и твердых тел, как они движутся?

Фронтальный опрос Какие из приведённых свойств принадлежат твёрдым телам, жидкостям и газам?

Фронтальный опрос Изменится ли объём газа, если его перекачать из баллона вместимостью 20 литров в баллон вместимость.40 литров?

Фронтальный опрос Есть ли такое вещество, у которого молекулы расположены на больших расстояниях, сильно притягиваются друг к другу и колеблются около определённых положений?

Фронтальный опрос В каком состоянии может находиться ртуть?

Как происходит процесс кипения ?

Испарение ли это?

Почему пузырьки лопаются вблизи поверхности жидкости

при температуре кипения?

Почему пузырьки

увеличиваются и

поднимаются вверх?

Почему слышен шум?

Температура

при кипении жидкости изменяется?

Почему в высокогорных районах приготовление пищи затруднительно?

Температура кипения жидкостей одинакова?

Какое количество теплоты необходимо, чтобы обратить жидкость массой 1кг в пар без изменения температуры.? А любой другой массы?

Изменяется ли температура

жидкости при

кипении?

Почему медицинские

инструменты стерилизуют

в автоклавах?

Кипение

Процесс кипения:

  • При поступлении теплоты увеличивается температура жидкости
  • Увеличивается объём пузырьков воздуха
  • На пузырёк действует сила Архимеда
  • Пузырёк всплывает и лопается, попадая в непрогретую часть жидкости
  • При равномерном нагревании жидкости, пузырёк доплывает и лопается на поверхности

Температура кипения

100оС

Температура при

которой жидкость кипит

Во время кипения температура жидкости не меняется

Кипение воды

Т, 0 С

100

t,с

8

15

18

Так происходит не только с водой, но и со всеми кипящими жидкостями

Температура кипения

Вещество

Температура, 0 С

водород

-253

кислород

-183

молоко

100

эфир

35

спирт

78

вода

100

ртуть

357

свинец

1740

медь

2567

Газы и твёрдые тела

  • Газы кипят при низких температурах.
  • Твёрдые тела кипят при высоких температурах.

Определение

Интенсивный переход жидкости в пар, происходящий с образованием пузырьков пара по всему объёму жидкости при определённой температуре, называют кипением.

Сходство и различие

Испарение

Процесс парообразования

Парообразование происходит с поверхности жидкости

Происходит при любой температуре

Кипение

Процесс парообразования

Парообразование происходит по всему объему жидкости

Происходит только при температуре кипения

Кипение в быту и промышленности

  • Кипение используется на электростанциях для выработки электричества.
  • Кипение используется в паровых установках и котельных для обогрева помещений.
  • Кипение используется в медицине для стерилизации инструментов.
  • Кипение используется для приготовления пищи.

Применение

Давление t 0C

скороварки

Для быстрого приготовления пищи

Давление увеличивается до 200 кПа

Вода кипит при температуре 120 0С

автоклавы

Используют для стерилизации медицинского инструмента. Вода кипит при температуре 3000С

  • Приготовление пищи в высокогорных районах будет не слишком успешным без скороварки

На высоте 7134 м (пик Ленина на Пальмире) вода кипит при 70 0 С

Применение

Давление t 0C

В сахарном производстве – выпаривание сахарного сиропа при пониженном давлении и

низкой температуре, чтобы сахар не подгорел

Температура кипения жидкости зависит:

1. От давления :

Чем больше давление , тем выше температура кипения.

2. От высоты над уровнем моря:

Чем выше тем меньше температура кипения.

Удельная теплота парообразования

Физическая величина, показывающая, какое количество теплоты необходимо, чтобы обратить жидкость массой 1кг в пар без изменения температуры.

Формула

Q

L= m

Формула

Q- Количество теплоты

L- Удельная теплота парообразования

m- масса

Q=Lm

Какое количество

теплоты потребуется, чтобы 20 кг. воды, взятой

при температуре 10 0с,

нагреть до кипения и

превратить в пар.

Какое количество энергии

требуется для обращения воды

массой 150 г в пар при

температуре 100 0С?

Как понимать, что

удельная

теплота

парообразования

воды равна

2,3 х 10

Какие явления

наблюдаются в жидкости

перед тем, как она

начинает кипеть?

Почему температура

кипения жидкостей

остается постоянной

независимо от

подводимого количества

теплоты?

6

Дж

кг

Решение задач

  • Разноуровневые самостоятельные и контрольные работы А. Кирик стр. 67 №1-6 устно, стр. 68 ср. уровень №1-3 письменно

Самостоятельная работа

Время выполнения: 15 минут

Критерии оценки:

  • «5» баллов – 5 верно выполненных заданий
  • «4» балла – 4 верно выполненных задания
  • «3» балла – 3 верно выполненных задания

Взаимопроверка Критерии оценки: «5» баллов – 5 верно выполненных заданий «4» балла – 4 верно выполненных задания «3» балла – 3 верно выполненных задания

Номер задания

 

Ответы

 

1

 

2

б

 

3

абвг

 

4

а

а

 

5

б

Домашнее задание

  • Создание мультимедийной презентации «Испарение и кипение», работа с конспектом лекции.
  • Решать загадки можно вечно. Вселенная ведь бесконечна. Спасибо всем нам за урок, А главное, чтоб был он впрок!

СПАСИБО ЗА ВНИМАНИЕ!

  Список использованных источников

  • Инфоурок / infourok.ru [Электронный ресурс] // Режим доступа: https://infourok.ru/otkritiy-urok-po-fizike-na-temu-isparenie-kipenie-nasischenniy-i-nenasischenniy-par-vlazhnost-vozduha-kriticheskoe-sostoyanie-ve-765573.html
  • Социальная сеть работников образования / nsportal.ru [Электронный ресурс] // Режим доступа: http://nsportal.ru/npo-spo/estestvennye-nauki/library/2012/06/14/nasyshchennyy-par-kipenie
  • Физика. 10 класс [Текст]: учебник для общеобразоват. учреждений с прил. на электронном носителе: базовый и профил. уровни / Г. Я. Мякишев, Б. Б. Буховцев, Н.Н. Соцкий.-11-е изд. — М. : Просвещение, 2003. – 336 с.
  • Физика-10. Разноуровневые самостоятельные и контрольные работы [Текст] / Л. А. Кирик ; Харьков: «Гимназия», 2002. – 192 с.
  • Электронное учебное пособие (приложение к учебнику Г.Я. Мякишев, Б. Б. Буховцев, Н. Н. Соцкий, Физика. 10 класс)

Реферат модельные представления о строении жидкостей, газов и кристаллов. Физика

Кинетическая энергия молекулы

В газе молекулы совершают свободное (изолированное от других молекул) движение, лишь время от времени сталкиваясь друг с другом или со стенками сосуда. До тех пор, пока молекула совершает свободное движение, у нее имеется только кинетическая энергия. Во время столкновения у молекул появляется и потенциальная энергия. Таким образом, полная энергия газа представляют сумму кинетической и потенциальной энергий ее молекул. Чем разреженное газ, тем больше молекул в каждый момент времени пребывает в состоянии свободного движения, имеющих только кинетическую энергию. Следовательно, при разрежении газа уменьшается доля потенциальной энергии в сравнении с кинетической.

Средняя кинетическая энергия молекулы пpи равновесии идеального газа обладает одной очень важной особенностью: в смеси различных газов средняя кинетическая энергия молекулы для различных компонентов смеси одна и та же.

Например, воздух представляет собой смесь газов. Средняя энергия молекулы воздуха для всех его компонентов пpи нормальных условиях, когда воздух еще можно рассматривать как идеальный газ, одинакова. Данное свойство идеальных газов может быть доказано на основании общих статистических соображений. Из него вытекает важное следствие: если два различных газа (в разных сосудах) находятся в тепловом равновесии друг с другом, то средние кинетические энергии их молекул одинаковы.

В газах обычно расстояние между молекулами и атомами значительно больше, чем размеры самих молекул, силы взаимодействия молекул не велики. Вследствие чего газ не имеет собственной формы и постоянного объема. Газ легко сжимается и может неограниченно расширяться. Молекулы газа движутся свободно (поступательно, могут вращаться), лишь иногда сталкиваясь с другими молекулами и стенками сосуда, в котором находится газ, причем движутся с очень большими скоростями.

Движение частиц в твердых телах

Строение твёрдых тел принципиально отлично от строения газов. В них межмолекулярные расстояния малы и потенциальная энергия молекул сравнима с кинетической. Атомы (или ионы, или целые молекулы) нельзя назвать неподвижными, они совершают беспорядочное колебательное движение около средних положений. Чем больше температура, тем больше энергия колебаний, а следовательно, и средняя амплитуда колебаний. Тепловыми колебаниями атомов объясняется и теплоемкость твёрдых тел. Рассмотрим подробнее движения частиц в кристаллических твердых телах. Весь кристалл в целом представляет собой очень сложную связанную колебательную систему. Отклонения атомов от средних положений невелики, и поэтому можно считать, что атомы подвергаются действию квазиупругих сил, подчиняющихся линейному закону Гука. Такие колебательные системы называются линейными.

Существует развитая математическая теория систем, подверженных линейным колебаниям. В ней доказана очень важная теорема, суть которой состоит в следующем. Если система совершает малые (линейные) взаимосвязанные колебания, то путем преобразования координат ее формально можно свести к системе независимых осцилляторов (у которых уравнения колебаний не зависят друг от друга). Система независимых осцилляторов ведет себя подобно идеальному газу в том смысле, что атомы последнего тоже можно рассматривать как независимые.

Именно используя представление о независимости атомов газа, мы приходим к закону Больцмана. Этот очень важный вывод представляет простую и надежную основу для всей теории твёрдого тела.

Закон Больцмана

Число осцилляторов с заданными параметрами (координаты и скорости) определяется так же, как и число молекул газа в заданном состоянии, по формуле:

Энергия осциллятора.

Закон Больцмана (1) в теории твёрдого тела не имеет ограничений, однако формула (2) для энергии осциллятора взята из классической механики. Пpи теоретическом рассмотрении твёрдых тел нужно опираться на квантовую механику, для которой характерна дискретность изменения энергии осциллятора. Дискретность энергии осциллятора становится несущественной только пpи достаточно высоких значениях его энергии. Это значит, что (2) можно пользоваться лишь пpи достаточно высоких температурах. Пpи высоких температурах твёрдого тела, близких к температуре плавления, из закона Больцмана вытекает закон равномерного распределения энергии по степеням свободы. Если в газах на каждую степень свободы в среднем приходится количество энергии, равное (1/2) kT, то у осциллятора одна степень свободы, кроме кинетической, имеет потенциальную энергию. Поэтому на одну степень свободы в твёрдом теле пpи достаточно высокой температуре приходится энергия, равная kT. Исходя из этого закона, нетрудно рассчитать полную внутреннюю энергию твердого тела, а вслед за ней и его теплоемкость. Моль твердого тела содержит NA атомов, а каждый атом имеет три степени свободы. Следовательно, в моле содержится 3 NA осцилляторов. Энергия моля твердого тела

а молярная теплоемкость твердого тела пpи достаточно высоких температурах

Опыт подтверждает этот закон.

Жидкости занимают промежуточное положение между газами и твердыми телами. Молекулы жидкости не расходятся на большие расстояния, и жидкость в обычных условиях сохраняет свой объем. Но в отличие от твердых тел молекулы не только совершают колебания, но и перескакивают с места на место, то есть совершают свободные движения. При повышении температуры жидкости кипят (существует так называемая температура кипения) и переходят в газ. При понижении температуры жидкости кристаллизуются и становятся твердыми веществами. Существует такая точка в поле температур, в которой граница между газом (насыщенным паром) жидкостью исчезает (критическая точка). Картина теплового движения молекул в жидкостях вблизи температуры затвердевания очень похожа на поведение молекул в твердых телах. К примеру, коэффициенты теплоемкости прочти совпадают. Так как теплоемкость вещества при плавлении изменяется слабо, то можно сделать вывод, что характер движения частиц в жидкости близок движению в твердом теле (при температуре плавления). При нагревании свойства жидкости постепенно изменяются, и она становится более похожа на газ. У жидкостей средняя кинетическая энергия частиц меньше потенциальной энергии их межмолекулярного взаимодействия. Энергия межмолекулярного взаимодействия в жидкости и твердых телах отличаются несущественно. Если сравнить теплоту плавления и теплоту испарения, то увидим, что при переходе из одного агрегатного состояния в другое теплота плавления существенно ниже, теплоты парообразования. Адекватное математическое описание структуры жидкости может быть дано лишь с помощью статистической физики. Например, если жидкость состоит из одинаковых сферических молекул, то ее структуру можно описать радиальной функцией распределения g(r), которая дает вероятность обнаружения какой-либо молекулы на расстоянии r от данной, выбранной в качестве точки отсчета.3\left(\frac{Дж}{кгК}\right).$

Задание: Объясните упрощённо с точки зрения физики процесс растворения соли (NaCl) в воде.

Основу современной теории растворов создал Д.И. Менделеев. Он установил, что при растворении протекают одновременно два процесса: физический — равномерное распределение частиц растворяемого вещества по всему объему раствора, и химический — взаимодействие растворителя с растворяемым веществом. Нас интересует физический процесс. Молекулы соли не разрушают молекулы воды. В этом случае нельзя было бы выпарить воду. Если бы молекулы соли присоединялись бы к молекулам воды — мы получали бы некое новое вещество. И внутрь молекул волы молекулы соли проникнуть не могут.

Между ионами Na+ и Cl— хлора и полярными молекулами воды возникает ионно-дипольная связь. Она оказывается прочнее, чем ионные связи в молекулах поваренной соли. В результате этого процесса связь между ионами, расположенными на поверхности кристаллов NaCl, ослабляется, ионы натрия и хлора отрываются от кристалла, а молекулы воды образуют вокруг них так называемые гидратные оболочки. Отделившиеся гидратированные ионы под влиянием теплового движения равномерно распределяются между молекулами растворителя.

Жидкость занимает по свойствам и строению промежуточное положение между газами и твердыми кристаллическими веществами. Поэтому обладает свойствами как газообразных, так и твердых веществ. В молекулярно-кинетической теории различные агрегатные состояния вещества связывают с различной степенью упорядоченности молекул. Для твердых тел наблюдается так называемый дальний порядок в расположении частиц, т.е. их упорядоченное расположение, повторяющееся на больших расстояниях. В жидкостях имеет место так называемый ближний порядок в расположении частиц, т.е. их упорядоченное расположение, повторяющееся на расстояниях, сравнимы с межатомными. При температурах, близких к температуре кристаллизации, структура жидкости близка к твердому телу. При высоких температурах, близких к температуре кипения, структура жидкости соответствует газообразному состоянию – практически все молекулы участвуют в хаотическом тепловом движении.

Жидкости, подобно твердым телам, обладают определенным объемом, а подобно газам, принимают форму сосуда, в котором они находятся. Молекулы газа практически не связаны между собой силами межмолекулярного взаимодействия, и в данном случае средняя энергия теплового движения молекул газа гораздо больше средней потенциальной энергии, обусловленной силами притяжения между ними, поэтому молекулы газа разлетаются в разные стороны и газ занимает предоставленный ему объем. В твердых и жидких телах силы притяжения между молекулами уже существенны и удерживают молекулы на определенном расстоянии друг от друга. В этом случае средняя энергия теплового движения молекул меньше средней потенциальной энергии, обусловленной силами межмолекулярного взаимодействия, и ее недостаточно для преодоления сил притяжения между молекулами, поэтому твердые тела и жидкости имеют определенный объем.

Давление в жидкостях с увеличением температуры и уменьшением объема возрастает весьма резко. Объемное расширение жидкостей гораздо меньше, чем паров и газов, так как более значительны силы, связывающие молекулы в жидкости; то же замечание касается теплового расширения.

Теплоемкости жидкостей обычно растут с температурой (хотя и незначительно). Отношение С р /С V практически равно единице.

Теория жидкости до настоящего времени полностью не развита. Разработка ряда проблем в исследовании сложных свойств жидкости принадлежит Я.И. Френкелю (1894–1952). Тепловое движение в жидкости он объяснял тем, что каждая молекула в течение некоторого времени колеблется около определенного положения равновесия, после чего скачком переходит в новое положение, отстоящее от исходного на расстоянии порядка межатомного. Таким образом, молекулы жидкости довольно медленно перемещаются по всей массе жидкости. С повышением температуры жидкости частота колебательного движения резко увеличивается, возрастает подвижность молекул.

На основе модели Френкеля можно объяснить некоторые отличительные особенности свойств жидкости. Так, жидкости даже вблизи критической температуры обладают гораздо большей вязкостью , чем газы, и вязкость с ростом температуры уменьшается (а не растет, как у газов). Объясняется это иным характером процесса передачи импульса: он передается молекулами, совершающими перескок из одного равновесного состояния в другое, а эти перескоки с ростом температуры существенно учащаются. Диффузия в жидкостях происходит только за счет перескоков молекул, и она происходит гораздо медленнее, чем в газах. Теплопроводность жидкостей обусловлена обменом кинетической энергии между частицами, колеблющимися около своих положений равновесия с различными амплитудами; резкие перескоки молекул заметной роли не играют. Механизм теплопроводности похож на механизм ее в газах. Характерной особенностью жидкости является ее способность иметь свободную поверхность (не ограниченную твердыми стенками).

В газах обычно расстояние между молекулами и атомами значительно больше размеров молекул, а силы притяжения очень малы. Поэтому газы не имеют собственной формы и постоянного объёма. Газы легко сжимаются, потому что силы отталкивания на больших расстояниях также малы. Газы обладают свойством неограниченно расширяться, заполняя весь предоставленный им объём. Молекулы газа движутся с очень большими скоростями, сталкиваются между собой, отскакивают друг от друга в разные стороны. Многочисленные удары молекул о стенки сосуда создают давление газа .

Движение молекул в жидкостях

В жидкостях молекулы не только колеблются около положения равновесия, но и совершают перескоки из одного положения равновесия в соседнее. Эти перескоки происходят периодически. Временной отрезок между такими перескоками получил название среднее время оседлой жизни (или среднее время релаксации ) и обозначается буквой?. Иными словами, время релаксации – это время колебаний около одного определённого положения равновесия. При комнатной температуре это время составляет в среднем 10 -11 с. Время одного колебания составляет 10 -12 …10 -13 с.

Время оседлой жизни уменьшается с повышением температуры. Расстояние между молекулами жидкости меньше размеров молекул, частицы расположены близко друг к другу, а межмолекулярное притяжение велико. Тем не менее, расположение молекул жидкости не является строго упорядоченным по всему объёму.

Жидкости, как и твёрдые тела, сохраняют свой объём, но не имеют собственной формы. Поэтому они принимают форму сосуда, в котором находятся. Жидкость обладает таким свойством, как текучесть . Благодаря этому свойству жидкость не сопротивляется изменению формы, мало сжимается, а её физические свойства одинаковы по всем направлениям внутри жидкости (изотропия жидкостей). Впервые характер молекулярного движения в жидкостях установил советский физик Яков Ильич Френкель (1894 – 1952).

Движение молекул в твёрдых телах

Молекулы и атомы твёрдого тела расположены в определённом порядке и образуют кристаллическую решётку . Такие твёрдые вещества называют кристаллическими. Атомы совершают колебательные движения около положения равновесия, а притяжение между ними очень велико. Поэтому твёрдые тела в обычных условиях сохраняют объём и имеют собственную форму.

Тема: Три состояния вещества

I вариант

I. Как расположены молекулы в твёрдых телах и как они движутся?

Молекулы расположены на расстояниях меньших размеров самих молекул и перемещаются свободно относительно друг друга. Молекулы расположены на больших расстояниях друг от друга (по сравнению с размерами молекул) и движутся беспорядочно. Молекулы расположены в строгом порядке и колеблются около определённых положений равновесия.

II. Какие из приведённых ниже свойств принадлежат газам?

Имеют определённый объём Занимают объём всего сосуда Принимают форму сосуда Мало сжимаются Легко поддаются сжатию

III. Изменится ли объём газа, если его перекачать из сосуда вместимостью 1 литр в сосуд вместимостью 2 литра?

IV. Молекулы расположены на больших расстояниях друг от друга (по отношению с размерами молекул), слабо взаимодействуют между собой, движутся хаотически. Какое это тело?

Газ Твёрдое тело Жидкость Такого тела нет

V. В каком состоянии может находиться сталь?

Только в твёрдом состоянии Только в жидком состоянии Только в газообразном Во всех трёх состояниях

Тема: Три состояния вещества

II вариант

I. Как расположены молекулы жидкостей и как они движутся?

Молекулы расположены на расстояниях, соизмеримых с размерами самих молекул, и перемещаются свободно относительно друг друга. Молекулы расположены на больших расстояниях (по сравнению с размерами молекул) друг от друга и движутся беспорядочно. Молекулы расположены в строгом порядке и колеблются около определённых положений равновесия.

II. Какие из приведённых свойств принадлежат газам?

Занимают весь предоставленный им объём Трудно сжимаются Имеют кристаллическое строение Легко сжимаются Не имеют собственной формы

III. В мензурке находится вода объёмом 100 см3. Её переливают в стакан вместимостью 200 см3. Изменится ли объём воды?

IV. Молекулы плотно упакованы, сильно притягиваются друг к другу, каждая молекула колеблется около определённого положения. Какое это тело?

Газ Жидкость Твёрдое тело Таких тел нет

V. В каком состоянии может находиться вода?

Только в жидком состоянии Только в газообразном состоянии Только в твёрдом состоянии Во всех трёх состояниях

Тема: Три состояния вещества

III вариант

I. Как расположены молекулы газов и как они движутся?

Молекулы расположены на расстояниях, меньших размеров самих молекул, и перемещаются свободно относительно друг друга. Молекулы расположены на расстояниях, во много раз больше размеров самих молекул, и движутся беспорядочно. Молекулы расположены в строгом порядке и колеблются около определённых положений.

II. Какие из приведённых свойств принадлежат твёрдым телам?

Трудно изменить форму Занимают весь предоставленный им объём Сохраняют постоянную форму Легко меняют форму Трудно сжимаются

III. Изменится ли объём газа, если его перекачать из баллона вместимостью 20 литров в баллон вместимость.40 литров?

Увеличится в 2 раза Уменьшится в 2 раза Не изменится

IV. Есть ли такое вещество, у которого молекулы расположены на больших расстояниях, сильно притягиваются друг к другу и колеблются около определённых положений?

Газ Жидкость Твёрдое тело Такого вещества не существует

V. В каком состоянии может находиться ртуть?

Только в жидком Только в твёрдом Только в газообразном Во всех трёх состояниях

Тема: Три состояния вещества

IV вариант

I. Ниже указано поведение молекул в твёрдых, жидких и газообразных телах. Что является общим для жидкостей и газов?

То, что молекулы расположены на расстояниях меньших размеров самих молекул и движутся свободно относительно друг друга То, что молекулы расположены на больших расстояниях друг от друга и движутся беспорядочно То, что молекулы движутся беспорядочно друг относительно друга То, что молекулы расположены в строгом порядке и колеблются около определённых положений

II. Какие из указанных свойств принадлежат твёрдым телам?

Имеют определённый объём Занимают объём всего сосуда Принимают форму сосуда Мало сжимаются Легко сжимаются

III. В бутылке находится вода объёмом 0,5 литра. Её переливают в колбу вместимостью 1 литр. Изменится ли объём воды?

Увеличится Уменьшится Не изменится

IV. Молекулы расположены так, что расстояние между ними меньше размеров самих молекул. Они сильно притягиваются друг к другу и перемещаются с места на место. Какое это тело?

Газ Жидкость Твёрдое тело

V. В каком состоянии может находиться спирт?

Только в твёрдом состоянии Только в жидком состоянии Только в газообразном состоянии Во всех трёх состояниях

Ответы к тестам

I вариант

II — 2 , 5

II вариант

II — 1, 4, 5

III вариант

II — 1, 3, 5

IV вариант

II — 1, 4

14.2: Факторы, влияющие на давление газа

  1. Последнее обновление
  2. Сохранить как PDF
  1. Факторы, влияющие на давление газа
    1. Количество газа
    2. Объем
    3. Температура
  2. Сводка
  3. Участники и атрибуты

Давление воздуха в баскетбольном мяче должно быть отрегулировано так, чтобы мяч отскакивал от правильная высота.Перед игрой судьи проверяют мяч, бросая его с высоты плеча и наблюдая, как далеко назад он отскакивает. Что бы сделал судья, если бы мяч не отскочил так далеко, как должен? Что бы он сделал, если бы он отскочил слишком высоко?

Давление внутри контейнера зависит от количества газа внутри контейнера. Если баскетбольный мяч не отскакивает достаточно высоко, судья может исправить ситуацию, используя ручной насос и добавив в мяч больше воздуха. И наоборот, если мяч подпрыгнет слишком высоко, он может выпустить немного воздуха из мяча.

Факторы, влияющие на давление газа

Вспомните из кинетико-молекулярной теории, что частицы газа движутся беспорядочно и по прямым линиям до тех пор, пока они упруго не столкнутся либо с другими частицами газа, либо с одной из стенок контейнера. Именно эти столкновения со стенками емкости определяют давление газа. Четыре переменных используются для описания состояния газа. Это давление \ (\ left (P \ right) \), объем \ (\ left (V \ right) \), температура \ (\ left (T \ right) \) и количество газа, измеренное количество родинок (\ left (n \ right) \).Мы рассмотрим отдельно, как объем, температура и количество газа влияют на давление в замкнутой газовой пробе.

Количество газа

На рисунке ниже показано, что происходит, когда воздух добавляется в жесткий контейнер . Жесткий контейнер — это контейнер, который не может расширяться или сжиматься. Стальная канистра является примером жесткого контейнера.

Рисунок \ (\ PageIndex {1} \): Увеличение давления с увеличением количества частиц газа. (CC BY-NC; CK-12)

Баллон слева содержит газ под определенным давлением.Присоединенный воздушный насос затем используется для удвоения количества газа в баллоне. Поскольку канистра не может расширяться, увеличившееся количество молекул воздуха будет сталкиваться с внутренними стенками канистры в два раза чаще, чем раньше. В результате давление внутри канистры увеличивается вдвое. Как вы можете себе представить, если в жесткий контейнер постоянно добавляется все больше и больше воздуха, он может в конечном итоге лопнуть. Уменьшение количества молекул в жестком контейнере дает обратный эффект — давление снижается.

Объем

На давление также влияет объем емкости. Если объем контейнера уменьшается, молекулы газа имеют меньше места для перемещения. В результате они будут чаще ударяться о стенки емкости, и давление возрастает.

На рисунке ниже показан баллон с газом, объем которого регулируется регулируемым поршнем. Слева поршень в основном вытянут, и манометр показывает определенное давление. Справа поршень был выдвинут так, что объем закрытой части емкости, в которой находится газ, был уменьшен вдвое.Давление газа увеличивается вдвое. Увеличение объема контейнера приведет к обратному эффекту, и давление газа уменьшится.

Рисунок \ (\ PageIndex {2} \): Уменьшение объема добываемого газа увеличение давления газа. (CC BY-NC; CK-12)

Температура

Было бы очень нецелесообразно ставить банку с супом над костром, не выпушив банку. При нагревании баллончик может взорваться. Кинетико-молекулярная теория объясняет почему. Воздух внутри жесткой банки с супом получает больше кинетической энергии за счет тепла, исходящего от костра.Кинетическая энергия заставляет молекулы воздуха двигаться быстрее, и они чаще и с большей силой ударяют о стенки контейнера. Повышение давления внутри может в конечном итоге превысить прочность банки, и она взорвется. Дополнительным фактором является то, что суп может закипеть, что поможет еще больше газа и большему давлению внутри банки.

На рисунке ниже слева показан баллон с газом комнатной температуры \ (\ left (300 \: \ text {K} \ right) \).Справа цилиндр нагревается до тех пор, пока температура по Кельвину не удвоится до \ (600 \: \ text {K} \). Кинетическая энергия молекул газа увеличивается, поэтому столкновения со стенками контейнера теперь более сильные, чем раньше. В результате давление газа увеличивается вдвое. Понижение температуры приведет к обратному эффекту, и давление заключенного газа уменьшится.

Рисунок \ (\ PageIndex {3} \): Повышение температуры приводит к увеличению давления. (CC BY-NC; CK-12)

Резюме

  • Увеличение количества молекул газа в контейнере того же объема увеличивает давление.
  • Уменьшение объема баллона увеличивает давление газа.
  • Повышение температуры газа в жестком контейнере увеличивает давление.

Авторы и авторство

  • Фонд CK-12 Шэрон Бьюик, Ричард Парсонс, Тереза ​​Форсайт, Шонна Робинсон и Жан Дюпон.

Как сделать P V T давление объем температура формула расчетов газа Закон Бойля Закон Чарльза Закон Гей-Люссака Идеальное поведение газа Примеры исправлений Примеры пересмотра GCSE химия физика KS4 наука A Уровень igcse revision notes

Док Брауна Химия KS4 science GCSE / IGCSE / AS Chemistry Revision Notes

Часть 2 Подробнее продвинутые темы об идеальном газе законы, расчеты, кинетическая теория моделей частиц (разделы 4a — 4b)

Более продвинутые идеи, связанные с газы расчет газового закона с использованием закона Бойля, закона Чарльза, закона Гей-Люссака, P1V1 / T1 = P2V2 / T2.Газовые законы, касающиеся давления, объема и температура очень важна как для GCSE, так и для химии A level

Основная теория частиц и свойства газов, жидкости и твердые тела, изменения состояния и решения описаны на Заметки GCSE / IGCSE по моделям частиц газов, жидкостей и твердых тел, описание и объяснение их свойств, и продвинутые студенты должны быть знакомы со ВСЕМ его содержанием перед изучением этой страницы ….

Субиндекс для части 2: Раздел 4 Поведение идеального газа и законы газа: Введение кинетическая теория частиц идеального газа * Температурная шкала Кельвина * 4а Закон Бойля * 4b.Закон Чарльза и Гей-Люссака и уравнение комбинированного закона газа * 4c. Уравнение идеального газа PV = nRT * 4d. Закон Дальтона о парциальных давлениях * 4e. Закон диффузии Грэма * 5а. Отклонения газов от идеального поведение и их причины * 5b. Уравнение состояния Ван-дер-Ваальса * 5c Коэффициенты сжимаемости * 5d В Критическая точка Критическая температура и критическое давление *

Для других расчетов см. страница «Индекс расчетов» включая определение родинки и Константа Авогадро

И молярный объем газа и объемные соотношения реагирующих газов.

Максвелл Больцмановское распределение кинетических энергий частиц обсуждается в КИНЕТИКА страницы.


Док Брауна Заметки о пересмотре химии: химия в основной школе, химия GCSE, химия IGCSE, уровень O & ~ Школьные курсы естественных наук для 8, 9 и 10 классов в США или их эквиваленты для детей от 14 до 16 лет студенты-естественники на государственных экзаменах по химии


4.Идеальное поведение газа и газовые законы

Введение кинетической модели частиц идеального газа

  • (продвинутый) кинетический Теория газов основана на следующих шести фундаментальных постулатах:

    1. Газы состоят из мельчайшие дискретные частицы (обычно молекулы).

    2. Частицы находятся в непрерывное хаотическое движение, движущееся по прямым линиям между очень частыми столкновениями с друг друга и стороны контейнера (примерно 10 9 / с).

    3. Обстрел стенки контейнера частицами вызывают явление, которое мы называем давлением (т.е. сила удара на единицу площади). Чем больше сила столкновения и чем чаще происходят столкновения, тем больше давление газа на поверхность контейнера.

    4. Столкновения идеально эластичный, т.е. без потерь энергии при столкновении из-за трения.

    5. На относительно низком уровне давления среднее расстояние между частицами велико по сравнению с диаметр частиц и, следовательно, межмолекулярные силы между частиц можно пренебречь.

    6. Средняя кинетическая энергия частиц прямо пропорциональна их абсолютной температуре по шкале Кельвина (K), т.е. KE (J) Т (К)

      • Это означает, что если вы нагреете газ, средняя кинетическая энергия частиц увеличивается, поэтому средняя скорость тоже увеличивается.

      • Температурная шкала Кельвина объяснено ниже.

  • Когда ведет себя газ Согласно этой модели, законы газа, описанные в разделах 4a — 4e, являются повиновался.

  • Однако в реальных газах все не так просто, и это неидеальное поведение обсуждается в разделе 5.


г. Температурная шкала Кельвина

В прошлом было проведено много измерений. сделано, чтобы исследовать, как (i) давление и объем данной массы изменяются при постоянная температура и как (ii) давление и объем газа фиксированной массы газ меняется в зависимости от температуры (см. два графика слева и справа).Это привело в формулировке законов газов, описанных в следующем разделе 4а. вдоль как использовать их в расчетах и ​​решении задач.

Однако перед этим, если вы посмотрите на эти два графика поведения газа при изменении давления или объема в зависимости от температуры, одна вещь становится ясной, когда линии графика экстраполируются обратно на x по оси они дают значение 273 o C. Это породило идею, что была минимально возможная температура 273 ° ° C и дальнейшие эксперименты подтверждали это снова и снова.При 273 o C все вещества твердые и с точки зрения теории кинетических частиц материи, при 273 o C частицы практически не двигаются т.е. ~ нет колебания атомов в твердом теле.

Следовательно, как и установлен Шкала Цельсия (шкала Цельсия), новая шкала температуры была предложена в наименьшее значение было 0 (известное как абсолютный ноль ), а не 273. Это называется шкалой Кельвина температуры или абсолютной температуры. Температурная шкала , обозначается единицей К .Между прочим, вы не говорите градусы Кельвина, как вы говорите градусы Цельсия, вы просто говорит Кельвин. Температурная шкала Кельвина также была разработана таким образом, чтобы 1 K изменение температуры или интервал, точно равняется 1 o C Изменение по Цельсию или интервал. Таким образом, вы можете легко переключаться между двумя температурными шкалами. простым расчетом

K = o C + 273 и o C = K 273 и это температура в К, которую вы должны использовать в расчетах по закону газа (4b.)

Некоторые примеры разработаны ниже и потренируйтесь в чтении термометра Цельсия, который вы используете в школе или колледж лаборатория!

7 o С

7 + 273 = 267 К

36 o С

36 + 273 = 309 К

77 o С

77 + 273 = 350 К

101.5 o С

101,5 + 273,0 = 374,5 К

132 o C

132 + 273 = 405 К

206 o С

206 + 273 = 479 К

Некоторые знакомые температуры указаны ниже. связь двух температурных шкал

абсолютный ноль Замораживание точка воды Кузов температура Кипячение точка воды
Цельсия шкала 273 о С 0 o С 37 o С 100 о С
Кельвин шкала 0 К 273 К 310 К 373 К

Кажется немного странным сказать, что ваше тело температура 310, поэтому всегда важно указывать и единицы измерения!

НАЧАЛО СТРАНИЦЫ


5.Модель частиц газа — движение и давление газа

  • Все частицы имеют массу , и их движение дает им кинетическую энергию и импульс.

  • Частицы в газе находятся в постоянном состоянии. случайное движение — случайное направление, различные скорости и кинетические энергии.

  • Когда быстро движутся частицы газа сталкиваются с поверхностью, их миллионы ударов создают силу, которую мы Измерить как давление газа — суммарную силу удара на единицу площади.

  • Частицы сталкиваются с контейнером поверхность полностью случайна и ударяется под любым углом, НО, эффект создать чистую силу под прямым углом к ​​поверхности — давление газа!

  • Чем больше количество столкновений на единицу площади поверхности, тем выше давление, если объем газа и температура остаются постоянными.

  • Если температура поддерживается постоянной и объем увеличился, удары стали более распространенными и менее частыми за единицу площади, поэтому давление газа снижается.

    • И наоборот, если газ сжатие в меньший объем при постоянной температуре, число ударов на единицу площади увеличивается, поэтому давление увеличивается.

    • Из измерений объемов и давление газов при постоянном давлении, числовой обратный закон может можно сформулировать — закон Бойля .

    • давление x объем = постоянная (при постоянной температуре)

    • pV = постоянная

    • p = давление в паскалях (Па), V = объем (м 3 )

    • Вы можете подключить два давления и два объема по простому уравнению

    • p 1 x V 1 = p 2 x V 2

    • где 1 представляет оригинал условия, и 2 конечная ситуация, если принудительное изменение p 1 или V 1 .

    • Примеры простого газа расчеты


Подробнее о расчетах давления и объема газа

4а. Закон Бойля для объема и газа давление

  • Теория частиц давления газа было объяснено в Части 1 поэтому в этом разделе основное внимание уделяется закону газа Расчеты с учетом давления и объема .

  • Закон Бойля гласит, что для данной массы газа при постоянной температуре ( o C или K) произведение давления, умноженное на объем постоянный.

  • p x V = постоянная

  • Следовательно, для начального значения p 1 и V 1 , которые изменяются на окончательные значения p 2 и V 2 , следующие уравнение применимо…

  • p 1 x V 1 = p 2 x V 2 (для фиксированного количества газа при постоянной температура)

  • или п 2 = p 1 x V 1 / V 2 или V 2 = p 1 x V 1 / p 2

  • График показывает, как давление и объем изменяются согласно закону Бойлса при двух разных температурах.

  • При более низких температурах объем и давление значения ниже (см. следующий раздел).

  • Вы можете использовать любой объем или единицы давления, которые вам нравятся, пока и и p, и V имеют те же единицы .

  • Используя теорию частиц и Простые арифметические значения для объясняют закон Бойлса .

    • Если газ сжимается вдвое Исходный объем: концентрация или плотность газа увеличиваются вдвое.Следовательно будет вдвое больше столкновений с поверхностью, что приведет к удвоению ударный эффект, т. е. удвоение давления.

    • Если объем газа увеличивается в три раза, концентрация уменьшается в том же размере. фактор, поэтому вероятность столкновения частиц со стенками контейнера составляет аналогично уменьшается, поэтому давление уменьшается в три раза.

  • Газы e.грамм. кислород для больниц, может быть хранятся под высоким давлением, что обеспечивает достаточно эффективное хранение. Потому что внутреннее давление в цилиндре намного больше, чем внешнее давление, при установке клапана большой объем газа может быть выпущен в поток медленно в контролируемых условиях для дыхания пациента.

  • Примеры расчетов по закону Бойля (предполагается постоянная температура)

  • Бывший.4 квартал 1

    • 240см 3 воздуха при давлении 100кПа в велосипедном насосе сжимается до объема 150см 3 .

    • Какое давление сжатый воздух в насосе?

    • p 1 x V 1 = p 2 x V 2 , перекомпоновка для увеличения для нового повышенное давление

    • п. 2 = p 1 x V 1 / V 2 = 100 x 240/150 = 160 кПа

  • Бывший.4 квартал 2

    • 10 м 3 бутана газ под давлением 1,2 атм требовалось хранить при давлении 6 атм. На какой объем нужно ли сжимать газ, чтобы получить необходимое давление хранения?

    • p 1 x V 1 = p 2 x V 2 , изменение масштаба для нового нижний объем

    • V 2 = p 1 x V 1 / p 2 = 1.2 x 10/6 = 2,0 м 3

  • Бывший. 4 кв. 3

    • 100 см 3 газовый шприц содержащий 80 см 3 газа, сжатого до 60 см 3 . Если атмосферное давление составляет 101325 Па, а температура остается постоянной, какое давление газа в шприце после сжатия.

    • p x V = постоянная

    • p 1 x V 1 = p 2 х В 2

    • п. 2 = p 1 x V 1 / V 2

    • п. 2 = 101325 x 80/60 = 135100 Па

  • Бывший.4 кв. 4

    • В стационаре давление газа в 100 дм 3 Баллон с кислородом составляет 5,52 атм (5-кратное атмосферное давление). Какой объем газ может медленно выделяться пациенту при выпуске его в атмосферное давление 1,01?

    • p x V = постоянная

    • В 2 = p 1 x V 1 / p 2

    • В 2 = p 1 x V 1 / p 2 = 5.52 x 100 / 1,01 = 546,5 дм 3

НАЧАЛО СТРАНИЦЫ


Расчеты 4б. Закон Чарльза / Закон Гей-Люссака для давления / объема и температура

и комбинированные уравнение газового закона

  • Теория частиц давления газа было объяснено в Части 1 , поэтому в этом разделе основное внимание уделяется закону газа. расчеты, включающие давление и объем и их изменение с температура.

  • Закон Чарльза / Гей-Люссака гласит, что для фиксированной массы газа

    • (i) объем газа напрямую пропорционально абсолютной температуре (K) при постоянном давлении

      • В = постоянная x T (правый график) или

      • В / Т = постоянная , или

      • В 1 / В 2 = T 1 / T 2 для условий, изменяющихся с 1 (начальное) до 2 (финал),

      • или V 1 / T 1 = V 2 / T 2 для постоянного давления

      • V 1 x T 2 = V 2 х Т 1

      • В 2 = В 1 х Т 2 / Т 1

      • или T 2 = T 1 x V 2 / V 1

      • Кинетические рассуждения о частицах — увеличение температура увеличивает кинетическую энергию молекул, давая более сильные столкновения, которые выталкивают (расширяют) газ при постоянном давление.

      • Обратите внимание, что графики экстраполируют назад к 0K (абсолютный ноль, шкала Кельвина) или -273 o C (Шкала Цельсия).

    • OR (ii) давление газа напрямую пропорционально абсолютной температуре (K) при постоянном объеме ,

      • p = постоянная x T (правый график), или

      • p / T = постоянная , или

      • п. 1 / п 2 = T 1 / T2 для условий, изменяющихся с 1 (начальное) до 2 (финал),

      • или p 1 / T 1 = p 2 / T 2 для постоянного объема

      • p 1 x Т 2 = Т 1 x п 2

      • p 2 = p 1 х Т 2 / Т 1

      • или T 2 = Т 1 x p 2 / p 1

      • Кинетические рассуждения о частицах — увеличение температура увеличивает кинетическую энергию молекул, давая более сильные столкновения, которые увеличивают давление, если объем ограниченный (постоянный).

      • Еще раз обратите внимание, что графики экстраполировать обратно к 0K (абсолютный ноль, шкала Кельвина) или -273 o C (Шкала Цельсия).

  • Во всех расчетах Абсолютная шкала температуры или шкала Кельвина должны использоваться для T ( K = o C + 273 ).

  • Если все законы описаны в 4a и 4b объединены, вы получите следующее общее выражение

  • p x V / T = постоянная (для заданная масса газа).

  • Это может быть выражено в обобщенная форма для расчетов на основе начальный набор условий 1 (1) изменение на новый и окончательный набор условий условия 2 (2) для данной массы газа, что дает уравнение для расчета комбинированного давления, объема, температуры …

  • p 1 x V 1 p 2 x V 2
    =
    Т 1 Т 2
  • Сокращенно ‘: p 1 V 1 / T 1 = p 2 V 2 / T 2
  • поэтому три перестановки для решения проблем с участием всех трех переменных:
    • p 2 = p 1 V 1 T 2 / В 2 Т 1
    • В 2 = p 1 V 1 T 2 / p 2 T 1
    • Т 2 = p 2 V 2 T 1 / p 1 V 1
  • Если одна переменная постоянна, перестановки для двух других переменных:

    • p 2 = p 1 V 1 / V 2 (при постоянном температура)
    • В 2 = В 1 Т 2 / Т 1 (при постоянном давлении)
    • Т 2 = p 2 T 1 / p 1 (при постоянном объеме)
  • Примечание :

    • Если температура постоянная, вы получаете Закон.

    • Если p или V постоянны, вы получить закон Чарльза / Гей-Люссака.

    • Вы можете использовать любой объем или единицы давления, которые вам нравятся, пока и p, или , оба V имеют те же единицы .

    • Графики p или V зависимости от температуры становятся недействительными, когда газ конденсируется в жидкость, НО при экстраполяции назад все строки кажутся происходящими из y = 0 (для p или V), x = 273 o C (для T).

    • Это было частью научных доказательств. это привело к убеждению, что 273 o C было минимально возможным температура, хотя теоретического верхнего предела нет вообще.

    • Это привело к разработке нового термодинамического абсолюта. температурная шкала или шкала Кельвина, которая начинается с ОК.

  • Примеры расчета PVT

  • Бывший.4b 1 квартал

    • Давление, оказываемое газ в герметичном контейнере составляет 100 кПа при 17 o C. Было обнаружено, что контейнер может протечь, если внутреннее давление превысит 120 кПа. Предполагая постоянный объем, при какой температуре в o C будет контейнер начать течь?

    • 17 o С + 273 = 290 К

    • п. 1 / т. 1 = P 2 / T 2

    • переставляем в масштаб до высших температура

    • T 2 = T 1 x p 2 / p 1

    • T 2 = 290 x 120/100 = 348 K или 348 273 = 75 o C когда контейнер может протечь

  • Бывший.4b 2 квартал

    • Баллон с пропаном газ при 20 o ° C оказывал давление 8,5 атмосфер. При воздействии на солнце она прогревается до 28 o С. Какое давление оказывает контейнерная сторона теперь опыт?

    • 20 o С = 273 + 20 = 293K, 28 o C = 273 + 28 = 301K

    • p 2 = p 1 х Т 2 / Т 1

    • p 2 = 8.5 x 301/293 = 8,73 атм

  • Бывший. 4b 3 квартал

    • Студент был исследование скорости реакции между гранулами известняка и различными концентрации соляной кислоты. Однако после целой серии экспериментов с разными концентрациями кислоты, не было времени заниматься последний запланированный эксперимент. Объем углекислого газа, собранный через 5 минут в 100 см 3 газовый шприц использовали для определения скорости реакции.Все эксперименты проводились за одно занятие при температуре 22 o C . кроме последнего. Это было сделано на следующем уроке, давая углеродный объем диоксида 47,0 см 3 через 5 минут, но при более высоком температура 27 o C (в градусах Кельвина назовите это T 1 , а другая температура T 2 ).

    • Сделать данные справедливый анализ, в идеале все объемы газа должны измеряться в одном и том же температура, но можно внести поправку для последнего эксперимента.

    • (a) Рассчитайте объем Объем газа 47,0 см 3 при 27 ° ° C, будет занимать при 22 ° ° C.

      • В 1 / В 2 = Т 1 / Т 2 поэтому V 2 = V 1 х Т 2 / Т 1

      • В 1 = 47.0 см 3 , T 1 = 273 + 27 = 300K, T 2 = 273 + 22 = 295 К

      • В 2 = 47,0 x 295/300 = 46,2 см 3

    • (b) Если температура был проигнорирован, каков% погрешности измерения скорости реакции?

      • Ошибка объема = 47.0 46,2 = +0,8 см 3 , следовательно ….

      • % ошибка = 0,8 x 100/47 = + 1,7% (так что вы бы переоценили скорость реакции без этой поправки)

      • % ошибки в объем будет таким же, как рассчитанный для ставки, например в см 3 / мин.

    • (c) Если Расчетное значение для 22 o C должно использоваться при расчете скорости анализ? и это еще один источник ошибок?

      • Теоретический расчетный объем газа для температуры 22 o ° C следует использовать для вычисляя скорость, это немного повысит точность, НО есть другая проблема!

      • Если реакция была к сожалению, выполняется при более высокой температуре (т.е.е. 27 o C) есть второй источник ошибок. При более высокой температуре реакция быстрее, поэтому вы обязательно получите больший объем газа, образованный за пять минут. Поэтому вы рассчитаете более высокую скорость реакции, например, в см 3 газа в минуту при 27 o C, что могло бы произойти / было бы измерено при 22 o C и поэтому несправедливое сравнение со всеми остальными результатами предыдущего урок.

      • Итак, хотя можно достаточно хорошо исправить ошибку объема из-за «расширенного» объема газа при при более высокой температуре объем газа все равно будет слишком большим из-за более высокая скорость реакции при 27 o C , и вас не так много может исправить эту ошибку, за исключением того, что повторите эксперимент при температуре 22 o ° C, что лучше всего делать в любом случае !

        • Обратите внимание, что если температура эксперимента скорости была слишком низкой по сравнению со всеми другими экспериментов, «двойная ошибка» повторится снова, но на этот раз измеренный объем газа и расчетная скорость / скорость реакции будут ниже чем ожидалось.

    • (d) Вам нужно сделать какая поправка на объем кислоты, добавленной в известняк? Объясни свой решение.

      • Коррекция не требуется для этого вообще . Хотя жидкости расширяются / сжимаются при нагревании / охлаждении, объем изменения намного меньше по сравнению с изменениями объема газа при той же температуре изменять. Это связано с относительно сильными межмолекулярными силами между жидкими молекул, которые практически отсутствуют в газах.

  • Бывший. 4b 4 квартал

    • 25 см 3 газа при 1.01 атм. при 25 o C был сжат до 15 см 3 при 35 o C.

    • Рассчитать финал давление газа.

    • п 1 = 1,01 атм, p 2 =?, V 1 = 25 см 3 , V 2 = 15 см 3 ,

    • Т1 = 25 + 273 = 298 К, Т2 = 35 + 273 = 308 К

    • (p 1 x V 1 ) / T 1 = (p 2 x V 2 ) / T 2

    • p 2 = (p 1 x V 1 x T 2 ) / (V 2 x T 1 )

    • p 2 = (1.01 x 25 x 308) / (15 x 298) = 1,74 атм

  • Бывший. 4b Q5

  • Бывший. 4b Q6

    • Топливные и воздушные газы в цилиндрах двигателя автомобиля 1200 см 3 перейти от 25 o C до сгорание и повышение до пиковой температуры 2100 o C после горение. Если нормальное атмосферное давление составляет 101 кПа, рассчитайте пиковое давление, достигнутое после сгорания.Хотя движение поршня изменяет громкость, (i) в качестве аргумента предположим, что громкость постоянный.

    • Т 1 = 25 + 273 = 298 К, ​​Т 2 = 2100 + 273 = 2373 К, P1 = 101 КПа

    • p / T = постоянная

    • п. 1 / п 2 = Т 1 / Т2

    • p 2 = p 1 х Т 2 / Т 1

    • p 2 = 101 x 2373/298 = 804 кПа

      • (ii) Чтобы быть более реалистичным, предположим, что начальный объем паров топлива и воздуха составлял 400 см3, теперь пересчитайте конечное давление.

      • Теперь вам нужно использовать полный PVT выражение.

      • (п 1 x V 1 ) / T 1 = (p 2 x V 2 ) / T 2
      • p 2 = (p 1 x V 1 x Т 2 ) / (В 2 x Т 1 )
      • p 2 = (101 x 400 x 2373) / (1200 x 298) = 268 кПа
  • Пр.Q7


Закон комбинированного газа уравнение (только для продвинутых студентов)

Существует уравнение, объединяющее оба Закон Бойля и закон Чарльза плюс моль газа.

Уравнение, известное как уравнение идеального газа , дается как:

pV = nRT

p = давление в паскали (единица измерения Па )

V = объем в кубических метров ( м 3 )

n = моль газа ( моль = масса в г / молекулярная масса газа M r )

R = постоянная идеального газа = 8.314 джоулей на кельвин на моль ( Дж · К -1 моль -1 )

T = температура в кельвинах ( K )

Вы должны преобразовать в эти единицы для правильного расчета с использованием pV = nRT

Подробнее о pV = nRT расчеты — более продвинутая химия стр.


НАЧАЛО СТРАНИЦЫ


ДРУГИЕ ПОЛЕЗНЫЕ СТРАНИЦЫ

Передовой примечания к расчетам газового закона, кинетические модельная теория ИДЕАЛЬНОГО ГАЗА и неидеальных газов

См. Также расчеты по газу

Моли и молярный объем газа, закон Авогадро

Объем реагирующего газа отношения, закон Авогадро Вычисления по закону Гей-Люссака

Все остальные страницы расчетов

  1. Что такое относительная атомная масса ?, относительная изотопная масса и расчет относительной атомной массы

  2. Расчет относительной формула / молекулярная масса соединения или молекулы элемента

  3. Закон сохранения массы и простые вычисления реагирующей массы

  4. Состав по процентной массе элементов в комплексе

  5. Эмпирическая формула и формула массы соединения из реагирующих масс (легкий старт, без родинок)

  6. Расчет соотношения реагирующих масс реагентов и продуктов из уравнений (Не используя моль) и краткое упоминание фактического процентного выхода и теоретического выхода, атомная экономика и определение массы по формуле

  7. Представляем родинки: связь между молями, массой и формульной массой основа расчета мольного отношения реагирующих веществ (относительно реагирующих масс и формулы масса)

  8. С помощью моль, чтобы вычислить эмпирическую формулу и вывести молекулярную формулу соединения / молекулы (исходя из реагирующих масс или% состава)

  9. Моли и молярный объем газа, закон Авогадро

  10. Объем реагирующего газа отношения, закон Авогадро и закон Гей-Люссака (соотношение газообразных продукты)

  11. Молярность, объемы и раствор концентрации (и схемы аппаратов)

  12. Как выполнить расчет объемного титрования e.грамм. кислотнощелочное титрование (и схемы аппаратов)

  13. Расчет продуктов электролиза (отрицательный катод и положительный анод)

  14. Прочие расчеты например % чистоты,% процентного содержания и теоретический выход, объемное титрование аппарат, разведение растворов (и схемы аппаратов), кристаллизационная вода, количество реагентов требуется, атом эконом

  15. Передача энергии при физических / химических изменениях, экзотермические / эндотермические реакции

  16. Расчеты по газу с учетом отношений PVT, Лоулз Бойля и Чарльза (эта страница)

  17. Расчеты радиоактивности и периода полураспада, включая датирующие материалы



НАЧАЛО СТРАНИЦЫ


Свойства газов

Свойства газов


Газы обладают тремя характерными свойствами: (1) легко сжимаются, (2) они расширяются, чтобы заполнить свои контейнеры, и (3) они занимают гораздо больше места, чем жидкости или твердые тела, из которых они образуются.

Сжимаемость

Двигатель внутреннего сгорания является хорошим примером легкости, с которой газы могут быть сжатым. В типичном четырехтактном двигателе поршень сначала вынимается из цилиндр для создания частичного вакуума, который втягивает смесь паров бензина и воздуха в цилиндр (см. рисунок ниже). Затем поршень проталкивается в цилиндр, сжимая смесь бензин / воздух до доли от ее первоначального объема.

Работу четырехтактного двигателя можно разделить на четыре цикла: впускной, ступени сжатия, мощности и выхлопа.

Отношение объема газа в баллоне после первого хода к его объему. после второго такта — степень сжатия двигателя. Современные автомобили бегут при степени сжатия около 9: 1, что означает наличие бензиновоздушной смеси в цилиндре сжимается в девять раз во втором такте.После того, как смесь бензин / воздух при сжатии свеча зажигания в верхней части цилиндра загорается, что приводит к взрыву. выталкивает поршень из цилиндра в третьем такте. Наконец, поршень толкается обратно в цилиндр в четвертом такте, очищая выхлопные газы.

Жидкости сжимать намного сложнее, чем газы. Их так сложно сжать, что гидравлические тормозные системы, используемые в большинстве автомобилей, работают по принципу практически никакого изменения объема тормозной жидкости при приложении к ней давления жидкость.Большинство твердых тел еще сложнее сжать. Единственные исключения относятся к редким класс смесей, включающий натуральный и синтетический каучук. Большинство резиновых мячей, которые кажутся легко сжимаются, например, ракетбол, наполнены воздухом, который сжимается, когда мяч сжимается.


Возможность расширения

Любой, кто заходил на кухню, где пекся хлеб, испытал на себе этот факт. что газы расширяются и заполняют свои контейнеры, так как воздух на кухне наполняется чудесные запахи.К сожалению, то же самое происходит, когда кто-то открывает гнилую яйца и характерный запах сероводорода (H 2 S) быстро распространяется через комнату. Поскольку газы расширяются и заполняют свои контейнеры, можно с уверенностью предположить, что объем газа равен объему его контейнера.


Объемы газов по сравнению с объемами жидкостей или твердые вещества

Разница между объемом газа и объемом жидкости или твердого тела от которые он формирует, можно проиллюстрировать следующими примерами.Один грамм жидкого кислорода при температуре кипения (-183 o ° C) имеет объем 0,894 мл. То же О 2 газ при 0 o C и атмосферном давлении имеет объем 700 мл, что составляет почти 800 раз больше. Аналогичные результаты получаются, когда объемы твердых тел и газов в сравнении. Один грамм твердого CO 2 имеет объем 0,641 мл. При 0 o C и атмосферного давления, такое же количество газа CO 2 имеет объем 556 мл, что более чем в 850 раз больше.Как правило, объем жидкости или твердого вещества увеличивается примерно в 800 раз, когда он образует газ.

Последствия этого огромного изменения объема часто используют для выполнения работы. В паровая машина, которая привела к промышленной революции, основана на том факте, что вода закипает с образованием газа (пара), имеющего гораздо больший объем. Таким образом, газ улетучивается. из контейнера, в котором он был произведен, и выходящий пар может сделать работай.Тот же принцип работает, когда динамит используется для взрыва камней. В 1867 г. Шведский химик Альфред Нобель обнаружил, что очень опасное жидкое взрывчатое вещество известно поскольку нитроглицерин может абсорбироваться глиной или опилками, чтобы произвести твердое вещество, которое более стабильный и, следовательно, более безопасный в использовании. Когда динамит взрывается, нитроглицерин разлагается с образованием смеси CO 2 , H 2 O, N 2 и O 2 газы.

4 C 3 H 5 N 3 O 9 ( л ) 12 CO 2 ( г ) + 10 H 2 O ( г ) + 6 N 2 ( г ) + O 2 ( г )

Потому что 29 моль газа производится на каждые четыре моля жидкости, которая разлагается, и каждый моль газа занимает объем примерно в 800 раз больше, чем моль жидкости, это реакция производит ударную волну, которая разрушает все, что находится поблизости.

То же явление происходит в гораздо меньших масштабах, когда мы готовим попкорн. Когда ядра попкорн нагревают в масле, жидкости внутри ядра превращаются в газы. Давление которые накапливаются внутри ядра, огромны, и в конечном итоге ядро ​​взрывается.


Давление в зависимости от силы

Объем газа — одно из его характерных свойств. Еще одна характеристика Свойство давление газа оказывает на окружающую среду.Многие из нас впервые столкнулись с давлением газа, когда мы поехали в соседний газ Станция для проверки давления в шинах велосипеда. В зависимости от типа велосипеда мы мы добавляли воздух в шины, пока манометр не показывал от 30 до 70 фунтов на квадратный дюйм (фунт / дюйм 2 или фунт / кв. дюйм). Два важных свойства давления могут быть получено из этого примера.

1. Давление газа увеличивается по мере добавления газа в контейнер.

2. Давление измеряется в единицах (например, фунт / дюйм 2 ), которые описывают силу оказываемого газом, разделенным на область , над которой эта сила распределены.

Первый вывод можно резюмировать в следующем соотношении, где P — давление газа; n — количество газа в баллоне.

п. n

Поскольку давление увеличивается по мере добавления газа в контейнер, P непосредственно пропорционально n .

Второй вывод описывает взаимосвязь между давлением и силой. Давление определяется как сила, приложенная к объекту, деленная на площадь, на которую действует сила распределены.

Разницу между давлением и силой можно проиллюстрировать аналогией, основанной на Гвоздь за 10 пенсов, молоток и кусок дерева, показанные на рисунке ниже. Отдыхая гвоздь на острие, и ударив молотком по головке, мы можем вбить гвоздь в дерево.Но что произойдет, если мы перевернем гвоздь и прижмем головку гвоздя к дерево? Если мы ударим по гвоздю с той же силой, мы не сможем заставить гвоздь воткнуться в дерево.

Когда мы попадаем в точку, сила этот удар наносят на очень небольшую площадь дерева, соприкасающуюся с острым концом гвоздь, и гвоздь легко входит в дерево. Но когда мы перевернем гвоздь и ударим На острие сила распределяется по гораздо большей площади.Сила сейчас распределяется по поверхности древесины, которая касается любой части шляпки гвоздя. Как в результате давление, прикладываемое к дереву, намного меньше, и гвоздь просто отскакивает от дерево.


Атмосферное давление

Что произойдет, если мы согнем длинную стеклянную трубку в форме буквы U, а затем осторожно наполнил одно плечо этой U-образной трубки водой, а другое — этиловым спиртом. алкоголь? Большинство людей ожидают, что высота столбиков жидкости в двух рукавах трубка должна быть такой же.Экспериментальным путем находим результаты, показанные на рисунке ниже. А 100-сантиметровая водная колонка уравновешивает 127-сантиметровую колонку этилового спирта независимо от диаметр стеклянной трубки.

Мы можем объяснить это наблюдение, сравнив плотность воды (1,00 г / см 3 ) и этиловый спирт (0,789 г / см 3 ). Столб воды высотой 100 см оказывает сильное воздействие. давление 100 грамм на квадратный сантиметр.

Колонна этилового спирта высотой 127 см оказывает такое же давление.

Потому что давление воды, давящей на одно плечо U-образной трубки, равно давление спирта, давящего на другой рукав трубки, система находится в баланс. Эта демонстрация дает основу для понимания того, как ртутный барометр может использоваться для измерения атмосферного давления.


Открытие барометра

В начале 1600-х годов Галилей утверждал, что всасывающие насосы могут забирать воду из ну из-за «силы вакуума» внутри насоса.После смерти Галилея итальянский математик и физик Евангелиста Торричелли (1608-1647) предложил другое объяснение. Он предположил, что воздух в нашей атмосфере имеет вес и что сила давления атмосферы на поверхность воды загоняет воду в всасывающий насос, когда он откачан.

В 1646 году Торричелли описал эксперимент, в котором стеклянная трубка длиной около метра была запечатанный с одного конца, наполненный ртутью, а затем перевернутый в емкость, наполненную ртутью, как показано на рисунке ниже.Некоторая, но не вся ртуть вытекла из стекла трубку в блюдо. Торричелли объяснил это тем, что ртуть стекает из стеклянную трубку до тех пор, пока сила столба ртути не надавит на внутри трубка точно уравновешивает силу атмосферы, давящую на поверхность жидкость за пределами трубки .

Торричелли предсказал, что высота ртутного столба будет меняться день ото дня. как давление атмосферы изменилось.Сегодня его прибор известен как барометр , от греческого baros , что означает «вес», потому что он буквально измеряет вес атмосферы. Повторные эксперименты показали, что среднее давление Атмосфера на уровне моря равна давлению столба ртути высотой 760 мм. Таким образом, стандартная единица давления, известная как атмосфера , определяется следующим образом.

1 атм = 760 мм рт. Ст.

Чтобы признать вклад Торричелли, некоторые ученые описывают давление в единицах «торр», которые определяются следующим образом.

1 торр = 1 мм рт. Ст.

Хотя химики все еще работают с единицами давления в атм или мм рт. принят в системе СИ. Единица давления в системе СИ — паскаль (Па). Отношения между одним стандартным атмосферным давлением и паскалем определяется следующим равенства.

1 атм = 101,325 Па = 101,325 кПа

Давление атмосферы можно продемонстрировать, подключив Банка емкостью 1 галлон к вакуумному насосу. Обычно давление газа внутри баллона уравновешивается. давление атмосферы на внешнюю сторону банки.Когда вакуумный насос при включении, однако, банка быстро разрушается при опорожнении.

Поверхность банки емкостью 1 галлон составляет около 250 дюймов 2 . При 14,7 фунт / дюйм 2 , это соответствует общей силе на поверхности банки около 3700 фунтов. Для сравнения можно отметить, что каждое из 18 колес грузовика весом 70000 фунтов несет всего около 3900 фунтов

Мы не чувствуем давления атмосферы, потому что давление внутри нашего тела уравновешивает давление газа в атмосфере.Последствия этого внутреннего давление было показано достаточно наглядно в нескольких фильмах. Прокол скафандра в вакууме космического пространства сразу приводит к разрыву тела, потому что есть ничего снаружи, чтобы уравновесить внутреннее давление тела.


Разница между давлением Газ и давление в зависимости от веса

Существует важное различие между давлением газа и другими примерами. давления, обсуждаемого в этом разделе.Давление со стороны женщины на высоких каблуках или 70000-фунтовый грузовик направлен. Например, грузовик оказывает все свое давление на поверхность под его колесами. Напротив, давление газа одинаково во всех направлениях. К Чтобы продемонстрировать это, мы можем наполнить стеклянный цилиндр водой и поставить на него стеклянную пластину. цилиндр. Когда мы переворачиваем цилиндр, тарелка не падает на пол, потому что давление воздуха за пределами цилиндра, толкающего вверх нижнюю часть пластины, равно больше, чем давление, оказываемое водой в цилиндре, давящей на пластину.Для создания давления, эквивалентного давлению, потребуется столб воды высотой 33,9 фута. давление газа в атмосфере.


Лекция 6 — Закон идеального газа, поднимающийся и опускающийся воздух

Лекция 6 — Закон идеального газа, поднимающийся и опускающийся воздух До этого момента мы думали о давлении как о определяется по весу воздуха над головой. Давление воздуха толкает вниз против земли на уровне моря с силой 14,7 фунта на квадрат дюйм.Если вы представите себе, как вес атмосферы давит на воздушный шар, сидя на земле, вы понимаете, что воздух в воздушном шаре отталкивает с той же силой. Воздух повсюду в атмосфере толкает вверх, вниз и в стороны.

Закон идеального газа уравнение — это еще один способ представления о давлении воздуха, своего рода вид в микроскопическом масштабе. Мы игнорируем атмосферу и сконцентрируйтесь только на воздухе внутри небольшого объема или воздушный шар или сверток * с воздухом. Мы собираемся «вывести» уравнение который показывает, как давление (P) зависит от определенных свойств воздуха спрятать воздушный шар.

* слово посылка просто означает небольшой объем воздуха.







Воздушные шары поднимаются (они же раковина), так же как и относительно теплый воздух при грозовом восходящем потоке (он теплее, чем воздух вокруг Это). Наоборот холодный воздух опускается. Поверхностные ветры вызванный нисходящим потоком грозы (как показано выше) может достигать скорости 100 миль в час и представляют собой серьезную погодную опасность.

Понимание закона идеального газа то первый шаг в объяснении того, что на самом деле вызывает подъем или опускание воздуха.

На втором шаге посмотрим по закону Чарльза, особая ситуация, связанная с законом идеального газа (воздух температура и плотность изменяются вместе таким образом, что давление внутри константы баллона). Тогда мы узнаем о вертикальные силы, действующие в воздухе (восходящие и направленная вниз сила) на этапе 3.

На рисунке выше показан важный момент: молекулы воздуха в Воздушные шары «наполненные воздухом» действительно занимают очень мало места. А воздушный шар, наполненный воздухом, действительно в основном пустое пространство.Это столкновения молекул воздуха с внутренними стенками воздушного шара которые удерживают воздушный шар в надутом состоянии.





А

Давление, создаваемое воздух молекулы внутри воздушного шара будут Во-первых, это зависит от количества молекул воздуха N в воздушном шаре. По мере того, как вы добавляете все больше и больше воздуха к чему-то вроде велосипедная шина давление увеличивается. Если здесь совсем не было молекул воздуха, не было бы никаких давление. Давление прямо пропорционально N — an увеличение N вызывает увеличение P.Если N удваивается, P также удваивается (при условии, что другие переменные в уравнении не изменяются).

In B
Давление воздуха внутри воздушного шара также зависит от размера воздушный шар. Давление обратно пропорционально объему, В . Если V увеличится вдвое, P упадет до 1/2 от исходного значения.

Примечание
Это можно поддерживать постоянное давление, изменяя N и V вместе правильным образом. Вот что происходит в эксперимент по концентрации кислорода, описанный на неделе 1.Кислород в градуированный цилиндр вступает в реакцию со стальной ватой с образованием ржавчины. Кислород удаляется из пробы воздуха, что является уменьшением N. Поскольку кислород удаляется, вода поднимается в цилиндр, уменьшая пробу воздуха объем. N и V уменьшаются в одинаковых относительных количествах и давление пробы воздуха остается постоянным. Если удалить 20% молекул воздуха, V уменьшится. до 20% от первоначального значения, и давление останется постоянным.



Часть C: Увеличение температура газа в воздушном шаре заставит молекулы газа двигаться быстрее.Они столкнутся со стенками воздушного шара чаще и отскок с большей силой. Оба будут увеличить давление. Не следует бросать баллончик с краской в огонь, потому что температура вызовет давление внутри может увеличиться, и банка может взорваться. Мы продемонстрируем влияние температуры на давление в классе в пятницу.

Удивительно, как объясняется в Части D, давление делает не зависит от массы молекулы. Давление не зависит от состава газ.Молекулы газа с большой массой будут двигаться медленнее, тем меньше массивные молекулы будут двигаться быстрее. Они оба столкнутся со стенками емкости с такой же силой.

На рисунке ниже показаны две формы закона идеального газа. В верх уравнение — это то, что мы только что вывели, а нижняя часть — чуть-чуть другая версия. Вы можете игнорировать константы k и R, если вы просто пытаетесь понять, как изменение одна из переменных повлияет на давление. Вам нужен только константы, когда вы выполняете вычисления с использованием чисел (которые мы не буду делать).



Закон Чарльза — частный случай закона идеального газа. Чарльз Лоу требует, чтобы давление в объеме воздуха оставалось постоянный. T, V и плотность могут изменяться, но они должны изменяться в способ, который сохраняет P постоянным. Вот что происходит в Атмосфера. Объемы воздуха в атмосфере могут свободно расширяться. или сжаться. Они делают это, чтобы поддерживать давление в воздушном объеме. константа ( давление внутри объема остается равным давлению воздуха вне объема).




Воздух в атмосфере ведет себя как воздух в воздушный шар. А воздушный шар может увеличиваться или уменьшаться в размерах в зависимости от давления воздуха внутри. Когда воздушный шар не становится больше или меньше, это означает сила внутри, которая выталкивает наружу, уравновешивается силой снаружи

Начнем с верхнего рисунка с воздуха внутри воздушного шара, который точно такой же, как воздух снаружи. Воздух внутри и снаружи были окрашены в зеленый цвет. Стрелки показывают, что давление воздуха внутри толкает наружу и давление воздуха воздух, окружающий воздушный шар, толкающийся внутрь, все тот же сила.

Далее прогреваем воздух в баллоне (рис. 2). Идеальный газ закон уравнение говорит нам, что давление воздуха в воздушном шаре увеличится. Увеличение составляет хотя и кратковременный.

Потому что давление внутри теперь больше (большие желтые стрелки) чем давление снаружи, воздушный шар расширится. Когда объем начинается для увеличения давление воздуха внутри воздушного шара будет снижаться. В конце концов воздушный шар расширится ровно настолько, чтобы давление внутри и снаружи снова в равновесии.Вы получаете воздушный шар теплого воздух низкой плотности, имеющий такое же давление, как и окружающий его воздух (Рис. 3)


Вы можете использовать те же рассуждения, чтобы пойми, что происходит, когда ты крут воздух в воздушном шаре.


Воздух внутри и снаружи то же на рис. 1. Охлаждение то воздух внутри воздушного шара на рис. 2 вызывает кратковременное падение внутри шара. давление (маленькие желтые стрелки) и создает давление дисбаланс. Более сильный наружный воздух давление сжимает баллон.


По мере уменьшения объема баллона давление внутри шара увеличивается. В конечном итоге он способен уравновесить внешний воздух давление. Вы получаете воздушный шар, наполненный холодным кайфом плотность воздуха.

Если нагреть воздух, он расширится и плотность будет уменьшаться до тех пор, пока давление внутри и снаружи посылки одинаковое.
Если охладить воздух, посылка сожмется, а плотность увеличится. пока давление не уравновесится.

Эти две ассоциации:

(я) теплый воздух = низкий воздух с плотностью
(ii) холодный воздух = воздух с высокой плотностью

важны и будут лот в течение оставшейся части семестр.


Вот краткое изложение закона Чарльза.

Если вы нагреете пучок воздуха, объем увеличится, а плотность уменьшится. Давление внутри посылки остается неизменным. Если охладить посылку воздухом его объем уменьшается, а его плотность увеличивается. Давление внутри посылка остается неизменной.



Чарльз Закон демонстрируется в классной версии этого курса путем погружения воздушный шар в жидкость азот.

Воздушный шар сжимается до практически нулевой объем, когда вытащили из жидкого азота.Он наполнен очень холодным высоким плотность воздуха в этой точке. В качестве воздушный шар согревает воздушный шар расширяется и плотность воздуха внутри баллон уменьшается. Объем и температура постоянно менялись таким образом, чтобы давление оставалось постоянным. В конце концов воздушный шар заканчивается обратно при комнатной температуре (если он не лопнет).


Теперь мы можем взглянуть на силы что может вызвать посылки воздуха подниматься или опускаться.



В основном все сводится к следующему — есть две силы действующий на воздушную подушку в атмосфере:
1.Гравитация тянет вниз. Сила силы тяжести зависит от от массы воздуха внутри посылка. Эта сила равна весу посылки
2. Разница давлений направлена ​​вверх. Этот сила вызванный воздухом снаружи посылка (воздух вокруг посылки). Давление снижается с увеличением увеличение высота. Давление воздуха внизу посылки толчок вверх немного сильнее, чем давление воздуха на верхушка воздушного шара, толкающая вниз.Общий эффект сила, направленная вверх.

Когда воздух внутри посылки точно такой же, как воздух за пределами, две силы равны по силе и уравновешиваются. Посылка является нейтрально буйный, не поднимается и не опускается.

Если заменить воздух внутри баллона на теплый с низкой плотностью воздух, это не будет так много весить. Сила тяжести слабее. В вверх сила перепада давления не меняется, так как определяется воздух за пределами воздушного шара, который не изменился и стал сильнее чем сила тяжести.Воздушный шар поднимется.

И наоборот, если воздух внутри представляет собой холодный воздух высокой плотности, он весит более. Гравитация сильнее разницы давлений вверх сила, и воздушный шар тонет.



ср может модифицировать то демонстрация, которую мы сделали ранее, чтобы продемонстрировать Закон Чарльза. В этом случае мы используем воздушные шары, наполненные гелием (или водородом). Гелий менее плотный чем воздух, даже когда гелий имеет ту же температуру, что и окружающий воздух. А Воздушный шар, наполненный гелием, не нуждается в подогреве, чтобы подняться.
Мы макаем наполненный гелием воздушный шар в жидкий азот для охлаждения Это и вызвать увеличение плотности гелия. Когда удаленный из жидкого азота баллон не поднимается, холодный газообразный гелий является плотнее окружающего воздуха (фиолетовые и синие воздушные шары в рисунок выше). Когда воздушный шар нагревается и расширяется его плотность гелия уменьшается. Воздушный шар в какой-то момент имеет то же самое плотности воздуха вокруг него (зеленый вверху) и нейтрально буйант. Со временем воздушный шар становится менее плотным, чем окружающий воздух (желтый) и плавает до потолка.

Что-то подобное происходит в Атмосфера.


При (1) солнечный свет, достигающий земли, поглощается и согревает земля. Это, в свою очередь, нагревает воздух, соприкасающийся с землей. (2) Как только этот воздух станет теплым и его плотность станет достаточно низкой, маленькие «капли» воздуха отделяются от слоя воздуха у земли и начинать подниматься. Их называют «термиками». (3) Поднимающийся воздух расширяется и остывает (это то, что мы еще не рассмотрели). Если остынет достаточно (до точки росы) облако будет становятся видимыми, как показано в пункте 4.Весь этот процесс называется свободная конвекция. Многие из летних гроз в южной Аризоне начни так.


Относительные силы направленная вниз сила тяжести и сила перепада давления вверх определить, поднимется или опускается порция воздуха. Архимед Закон — это еще один способ понять эту тему.
Галлон вода весит около 8 фунтов (фунтов).

Если вы погрузите кувшин с водой объемом 1 галлон в бассейн, кувшин становится для всех намерений и целей невесомым.Архимед’ Закон (см. Рисунок ниже, со стр. 53a в фотокопиях ClassNotes) объясняет, почему это правда.




Сила восходящего движения действительно просто другое название для сила перепада давления, покрытая ранее сегодня (более высокое давление толкать на бутылку и низкое давление в верхней части толкает вниз, что приводит к чистая направленная вверх сила). Бутылка на 1 галлон вытеснит 1 галлон вода в бассейне. Один галлон бассейна вода весит 8 фунтов. Сила восходящего движения составит 8 фунтов, то же, что и сила тяжести, действующая на кувшин.Два силы равны и противоположны.

Теперь мы представляем, как выливаем всю воду и наполняем 1 галлон. кувшин с воздухом. Воздух примерно в 1000 раз менее плотный, чем вода; по сравнению с поливать кувшин практически ничего не будет весить.



Если вы погрузите кувшин в бассейн он вытеснит 1 галлон воды и снова испытайте 8-фунтовую восходящую силу буйанта. С тех пор если нет направленной вниз силы, кувшин будет плавать.

Один галлон песка (который примерно в 1,5 раза плотнее воды) кувшин будет весить 12 фунтов.


Кувшин с песком утонет, потому что направленная вниз сила больше чем восходящая сила.

Вы можете подытожить все это, сказав что-нибудь менее плотное чем вода будет плавать в воде, все, что плотнее воды, будет плавают в воде.

То же самое относится и к воздуху в атмосфере.



Воздух менее плотный (более теплый) чем воздух вокруг него будет подъем. Воздух, который более плотный (холодный), чем воздух вокруг него, будет опускаться.


Красочная демонстрация того, насколько малы различия в плотности может определить, плавает ли объект или тонет.

Банки как обычного, так и диетического пепси помещают в наполненные стаканы. с водой (также можно использовать кока-колу и диетическую колу).

Обе банки изготовлены из алюминия, плотность которого почти втрое выше. выше воды. Сам напиток в основном состоит из воды. В в обычном пепси также много кукурузного сиропа с высоким содержанием фруктозы. Пепси нет. Смесь воды и кукурузного сиропа имеет густоту больше, чем просто воды.Также есть немного воздуха (или, возможно, углекислого газа) в каждой банке.

Средняя плотность банки обычной Пепси (вода и кукурузный сироп + алюминий + воздух) оказывается немного больше, чем плотность воды. Средняя плотность банки диеты Пепси (вода + алюминий + воздух) немного меньше плотности воды.

В некоторых отношениях люди в плавательных бассейнах подобны банкам из-под обычной и Диетическая содовая. Некоторые люди плавают (они немного менее плотные, чем вода), другие люди тонут (чуть плотнее воды).


Многие люди могут наполнить легкие воздухом и заставить себя плавать, или они могут опустошить свои легкие и заставить себя тонуть. Люди должны иметь плотность примерно такую ​​же, как у воды.


Законы об идеальном газе | Идеальные газы

7.2 Закон об идеальных газах (ESBNV)

Есть несколько законов, объясняющих поведение идеальных газов. Первые три, которые мы рассмотрим, применяются при очень строгих условиях. Затем эти законы объединяются, чтобы сформировать общее уравнение газа и уравнение идеального газа.

Прежде чем мы начнем изучать эти законы, нам нужно взглянуть на некоторые общие преобразования для единиц.

В следующей таблице приведены единицы СИ. {- 1} $} \)

Температура (\ (\ text {K} \))

Кельвин (\ (\ text {K} \))

\ (\ text {K} = \ text {℃} + \ text {273} \)

Таблица 7.{3} $} \).

Закон Бойля: Давление и объем закрытого газа (ESBNW)

Если вы когда-либо пытались протолкнуть поршень шприца или велосипедного насоса, закрывая отверстие пальцем, вы увидите закон Бойля в действии! Следующий эксперимент позволит вам увидеть этот закон в действии.

Закон Бойля

Давление фиксированного количества газа обратно пропорционально его объему, пока температура остается постоянной.

Неформальный эксперимент включен здесь. Это необходимо для проверки закона Бойля и проверки закона Чарльза. Этот эксперимент разделен на две части, каждая из которых приведена в соответствующем разделе книги. Для эксперимента по закону Бойля вам понадобится манометр, \ (\ text {10} \) \ (\ text {ml} \) шприц, \ (\ text {3} \) \ (\ text {cm} \ ) силиконовая трубка и чаша для воды или аппарат закона Бойля. Учащиеся построят график своих результатов и используют их, чтобы определить, был ли подтвержден закон Бойля.Они должны получить изогнутую линию для построения графика зависимости давления (\ (x \) — ось) от объема (\ (y \) — ось) и прямую линию при построении графика зависимости давления от обратной величины объема (\ (\ frac {\ текст {1}} {V} \)).

Закон Бойля

Цель

Чтобы проверить закон Бойля.

Аппарат

Аппарат закона Бойля (шприц или велосипедный насос, прикрепленный к манометру).

Метод

  1. Используйте насос для наполнения шприца (или стеклянной трубки) до тех пор, пока манометр не покажет максимальное значение.Обратите внимание на объем и давление.

  2. Медленно выпустите воздух, пока давление не упадет примерно на \ (\ text {20} \) единиц (единицы будут зависеть от того, что измеряет ваш манометр, например, \ (\ text {kPa} \)).

  3. Дайте системе стабилизироваться примерно на \ (\ text {2} \) \ (\ text {min} \), а затем прочтите том.

  4. Повторите два вышеуказанных шага, пока не получите шесть пар показаний давление-объем.

Результаты

Запишите результаты в следующую таблицу. Помните, что ваши единицы давления и объема будут определяться устройством, которое вы используете.

Что происходит с объемом при понижении давления?

Изобразите ваши результаты в виде графика зависимости давления от объема (другими словами, нанесите давление на ось x, а объем на ось y). Давление — это независимая переменная, которую мы изменяем, чтобы увидеть, что происходит с объемом.

Изобразите свои результаты в виде графика зависимости давления от обратной величины объема (другими словами, \ (\ frac {\ text {1}} {V} \), чтобы для каждого показания объема вы вычислили значение \ (\ текст {1} \), разделенный на объем чтения).

Что вы заметили в каждом графике?

Заключение

Если объем газа уменьшается, давление газа увеличивается. Если объем газа увеличивается, давление снижается. Эти результаты подтверждают закон Бойля.

В приведенном выше эксперименте объем газа уменьшался, когда давление увеличивалось, и объем увеличивался, когда давление уменьшалось.Это называется обратной зависимостью (или более-менее зависимостью). Обратное соотношение между давлением и объемом показано на рисунке 7.3.

Рисунок 7.3: График, показывающий обратную зависимость между давлением и объемом.

Можете ли вы использовать кинетическую теорию газов, чтобы объяснить эту обратную зависимость между давлением и объемом газа? Давай подумаем. Если вы уменьшите объем газа, это означает, что теперь такое же количество частиц газа будет гораздо чаще контактировать друг с другом и со стенками контейнера.Но мы сказали, что давление является мерой столкновений частиц газа друг с другом и со сторонами контейнера, в котором они находятся. Таким образом, если объем уменьшается, количество столкновений увеличивается, и поэтому давление естественно увеличится. Обратное верно, если объем газа увеличен. Теперь частицы газа сталкиваются реже и давление упадет.

Роберт Бойль — ученый, которому приписывают открытие, что давление и объем пробы газа обратно пропорциональны .Это можно увидеть, построив график зависимости давления от объема, обратного величине. Когда значения нанесены на график, график представляет собой прямую линию. Эта взаимосвязь показана на рисунке 7.4.

Рисунок 7.4: График зависимости давления от величины, обратной величине, представляет собой прямую линию. Это показывает, что давление и объем обратно пропорциональны.

Мы только что видели, что давление газа обратно пропорционально объему газа, при условии, что температура остается той же.Мы можем символически записать это отношение как:

\ [p ∝ \ frac {\ text {1}} {V} \]

где \ (∝ \) означает пропорциональный, а мы пишем \ (\ frac {\ text {1}} {V} \), чтобы показать, что пропорциональность обратная.

Это уравнение также можно записать в следующем виде:

\ [p = \ frac {k} {V} \]

где \ (k \) — постоянная пропорциональности. Если мы изменим это уравнение, мы можем сказать, что:

\ [pV = k \]

Это уравнение означает, что, если температура и количество газа постоянны, умножение любых значений давления и объема для фиксированного количества газа всегда дает одно и то же значение (\ (k \)).Например: \ begin {align *} p_ {1} V_ {1} & = k \\ p_ {2} V_ {2} & = k \ end {align *}

, где нижние индексы \ (\ text {1} \) и \ (\ text {2} \) относятся к двум парам показаний давления и объема для одинаковой массы любого газа при одинаковой температуре.

Отсюда мы можем сказать, что:

\ [\ в коробке {p_ {1} V_ {1} = p_ {2} V_ {2}} \]

И мы также можем обобщить это, чтобы сказать, что: \ begin {align *} p_ {1} V_ {1} & = p_ {2} V_ {2} \\ & = p_ {3} V_ {3} \\ & = p_ {n} V_ {n} \ end {align *}

Другими словами, мы можем использовать любые две пары показаний, это не обязательно должны быть первое и второе показания, это могут быть первое и третье показания или второе и пятое показания.{3} $} \ end {align *}

Помните, что закон Бойля требует двух условий. Во-первых, количество газа должно оставаться постоянным. Если вы позволите небольшому количеству воздуха выйти из контейнера, в котором он заключен, давление газа будет уменьшаться вместе с объемом, и соотношение обратной пропорции нарушится. Во-вторых, температура должна оставаться постоянной. Охлаждающее или нагретое вещество обычно заставляет его сжиматься или расширяться, или давление уменьшаться или увеличиваться.В эксперименте, если бы мы нагревали газ, он расширился бы, и вам потребовалось бы приложить большую силу, чтобы удерживать плунжер в заданном положении. Опять же, пропорциональность будет нарушена.

Знаете ли вы, что механизмы, участвующие в дыхании , также связаны с законом Бойля? Чуть ниже легких находится мышца, называемая диафрагмой . Когда человек вдыхает, диафрагма опускается вниз и становится более «плоской», так что объем легких может увеличиваться. Когда объем легких увеличивается , давление в легких уменьшается (закон Бойля).Поскольку воздух всегда перемещается из областей с высоким давлением в области с более низким давлением, воздух теперь будет втягиваться в легкие, потому что давление воздуха за пределами тела выше, чем давление в легких . Противоположный процесс происходит, когда человек выдыхает. Теперь диафрагма движется вверх и вызывает уменьшение объема легких до . {3} $} \)

Проверьте свой ответ

Закон Бойля гласит, что давление обратно пропорционально объему.Поскольку давление уменьшилось, объем должен увеличиться. Наш ответ для объема больше, чем исходный объем, поэтому наш ответ разумный.

Нет необходимости переводить в единицы СИ для закона Бойля. Для преобразования давления и объема в разные единицы требуется умножение на . Если бы вы изменили единицы в приведенном выше уравнении, это потребовало бы умножения с обеих сторон уравнения, и поэтому преобразования компенсировали бы друг друга. Однако, хотя единиц SI и использовать не обязательно, вы должны убедиться, что для каждой переменной вы используете одинаковые единицы во всем уравнении.Это неверно для некоторых расчетов, которые мы сделаем на более позднем этапе, когда должны использоваться единицы СИ .

Рабочий пример 2: закон Бойля

Объем пробы газа увеличен с \ (\ text {2,5} \) \ (\ text {L} \) до \ (\ text {2,8} \) \ (\ text {L} \) при поддержании постоянной температуры. Каково конечное давление газа при данных условиях объема, если начальное давление равно \ (\ text {695} \) \ (\ text {Pa} \)?

Запишите всю информацию, которую вы знаете о газе.

\ begin {align *} V_ {1} & = \ text {2,5} \ text {L} \\ V_ {2} & = \ text {2,8} \ text {L} \\ p_ {1} & = \ text {695} \ text {Pa} \\ p_ {2} & =? \ end {align *}

Выберите соответствующее уравнение газового закона для вычисления неизвестной переменной.

Образец газа имеет постоянную температуру, поэтому мы можем использовать закон Бойля: \ [p_ {2} V_ {2} = p_ {1} V_ {1} \]

Подставьте известные значения в уравнение, убедившись, что единицы измерения для каждой переменной равны

и .Рассчитайте неизвестную переменную.

\ begin {align *} (\ text {2,8}) p_ {2} & = (\ text {695}) (\ text {2,5}) \\ (\ text {2,8}) p_ {2} & = \ text {1 737,5} \\ p_ {2} & = \ text {620,5} \ text {кПа} \ end {align *}

Давление газа в объеме \ (\ text {2,8} \) \ (\ text {L} \) равно \ (\ text {620,5} \) \ (\ text {kPa} \ )

Проверьте свой ответ

Закон Бойля гласит, что давление обратно пропорционально объему. {3} $} \) воздуха под давлением \ (\ text {90} \) \ (\ text {кПа } \).{3} $} \ end {выровнять *}

Постройте график зависимости давления (p) от объема (V). Объем будет по оси абсцисс, а давление — по оси ординат. Опишите отношения, которые вы видите.

Постройте график зависимости p от \ (\ dfrac {1} {V} \). Опишите отношения, которые вы видите.

График представляет собой прямую линию. Эта прямая линия показывает обратную зависимость между давлением и объемом.

Подтверждают ли ваши результаты закон Бойля? Поясните свой ответ.

График зависимости p от \ (\ frac {1} {V} \) дает прямую линию, которая показывает, что давление обратно пропорционально объему. Это ожидаемый результат закона Бойля.

Рассчитайте ожидаемое конечное давление, используя начальное давление и объем, а также конечный объем.

\ begin {align *} p_ {2} V_ {2} & = p_ {1} V_ {1} \\ (\ text {3}) p_ {2} & = (\ text {650}) (\ text {1}) \\ \ text {3} p_ {2} & = \ text {650} \\ p_ {2} & = \ text {217} \ text {Па} \ end {выровнять *}

Кто правильно соблюдал закон Бойля и почему?

Из приведенного выше расчета мы видим, что Жюстин имеет правильное давление, предсказанное законом Бойля.Масоаби не получил ожидаемых результатов.

Жюстин поддерживала постоянную температуру и, таким образом, правильно повторила закон Бойля. Масоаби изменил температуру для второй части эксперимента и поэтому не повторил закон Бойля (на самом деле она повторила другой газовый закон).

Закон Чарльза: Объем и температура замкнутого газа (ESBNX)

Закон Чарльза описывает взаимосвязь между объемом и температурой газа.Закон был впервые опубликован Жозефом Луи Гей-Люссаком в 1802 году, но он сослался на неопубликованную работу Жака Шарля примерно 1787 года. Этот закон гласит, что при постоянном давлении объем данной массы идеального газа увеличивается или уменьшается на то же самое. фактор по мере того, как его температура (в Кельвинах) увеличивается или уменьшается. Другими словами, температура и объем прямо пропорциональны .

Закон Чарльза

Объем замкнутой пробы газа прямо пропорционален ее температуре по Кельвину при условии, что давление и количество газа поддерживаются постоянными.

Неформальный эксперимент включен здесь. Это необходимо для проверки закона Бойля и закона Чарльза (см. Примечания к закону Бойля). Во второй части эксперимента учащиеся проверят закон Чарльза. Вам понадобятся стеклянные бутылки, воздушные шары, мензурки или кастрюли, вода, плита. Учащиеся помещают воздушный шарик в горлышко бутылки, затем помещают его в стакан или кастрюлю с водой и смотрят, что происходит. Они также могут поместить бутылку с воздушным шариком в морозильную камеру и посмотреть, что произойдет.

Закон Чарльза

Цель

Чтобы продемонстрировать закон Чарльза.

Аппарат

стеклянная бутылка (например, пустая стеклянная бутылка из-под кокса), воздушный шар, химический стакан или горшок, вода, горячая плита

Метод

  1. Поместите баллон над отверстием пустой бутылки.

  2. Наполните стакан или кастрюлю водой и поставьте на плиту.

  3. Поставьте бутылку в стакан или кастрюлю и включите конфорку.

  4. Посмотрите, что происходит с воздушным шаром.

Результаты

Вы должны увидеть, что воздушный шар начинает расширяться. Когда воздух внутри бутылки нагревается, давление также увеличивается, что приводит к увеличению объема. Поскольку объем стеклянной бутылки не может увеличиваться, воздух попадает в воздушный шар, заставляя его расширяться.

Заключение

Температура и объем газа напрямую связаны друг с другом.По мере увеличения одного увеличивается и другое.

Вы также можете увидеть это, если поместите баллон и бутылку в морозильную камеру. После непродолжительного пребывания в морозильной камере воздушный шар сожмется.

Математически взаимосвязь между температурой и давлением может быть представлена ​​следующим образом:

\ [V ∝ T \]

Другими словами, объем прямо пропорционален температуре.

Или, заменив символ пропорциональности на коэффициент пропорциональности (\ (k \)):

\ [V = kT \]

Если уравнение изменить, то: \ [\ frac {V} {T} = k \]

или: \ begin {align *} \ frac {V_ {1}} {T_ {1}} & = k \\ \ frac {V_ {2}} {T_ {2}} & = k \\ \ frac {V_ {n}} {T_ {n}} & = k \ end {align *}

Итак, мы можем сказать, что:

\ [\ в коробке {\ frac {V_ {1}} {T_ {1}} = \ frac {V_ {2}} {T_ {2}}} \]

Уравнение, связывающее объем и температуру, дает прямолинейный график.

Однако, если мы построим этот график, используя температурную шкалу Цельсия (то есть используя \ (\ text {℃} \)), нулевая точка температуры не будет соответствовать нулевой точке объема. Когда объем равен нулю, температура на самом деле равна \ (- \ text {273} \) \ (\ text {℃} \) (рис. 7.5).

Рисунок 7.5: Соотношение между объемом и температурой, показанное на шкале температур по Цельсию.

Вместо нее необходимо использовать новую шкалу температуры Кельвина .Поскольку ноль на шкале Цельсия соответствует температуре Кельвина \ (- \ text {273} \) \ (\ text {℃} \), можно сказать, что:

\ (\ text {Температура по Кельвину (} T_ {K} \ text {)} = \ text {Температура по Цельсию (} T_ {C} \ text {)} + \ text {273} \)

Мы можем написать:

\ (T_ {K} = T_ {C} + \ text {273} \)

или

\ (T_ {C} = T_ {K} — \ text {273} \)

Теперь мы можем построить график зависимости температуры от объема по шкале Кельвина. Это показано на рисунке 7.6.

Рисунок 7.6: Объем газа прямо пропорционален его температуре, если давление газа постоянно.

Можете ли вы объяснить закон Чарльза с точки зрения кинетической теории газов? Когда температура газа увеличивается, увеличивается и средняя скорость его молекул. Молекулы сталкиваются со стенками емкости чаще и с большим ударом. Эти столкновения отодвигают стены, так что газ занимает больший объем, чем вначале. Мы видели это на первой демонстрации.Поскольку стеклянная бутылка не могла расшириться, вместо этого газ вытолкнул воздушный шар.

Рабочий пример 3: закон Карла

При температуре \ (\ text {298} \) \ (\ text {K} \) определенное количество газа \ (\ text {CO} _ {2} \) занимает объем \ (\ text {6} \) \ (\ text {L} \). Какой температуры будет газ, если его объем уменьшится до \ (\ text {5,5} \) \ (\ text {L} \)? Давление остается постоянным.

Запишите всю информацию, которую вы знаете о газе.

\ begin {align *} V_ {1} & = \ text {6} \ text {L} \\ V_ {2} & = \ text {5,5} \ text {L} \\ T_ {1} & = \ text {298} \ text {K} \\ T_ {2} & =? \ end {align *}

При необходимости преобразуйте известные значения в единицы СИ.

Температурные данные уже указаны в градусах Кельвина, поэтому преобразование не требуется.

Выберите соответствующее уравнение закона газа, которое позволит вам вычислить неизвестную переменную.

Давление поддерживается постоянным при изменении объема и температуры. Количество газа также остается постоянным, поэтому мы можем использовать закон Чарльза: \ [\ frac {V_ {1}} {T_ {1}} = \ frac {V_ {2}} {T_ {2}} \]

Подставьте известные значения в уравнение. Рассчитайте неизвестную переменную.

\ begin {align *} \ frac {\ text {6}} {\ text {298}} & = \ frac {\ text {5,5}} {T_ {2}} \\ \ text {0,0201} \ ldots & = \ frac {\ text {5,5}} {T_ {2}} \\ (\ text {0,0201} \ ldots) T_ {2} & = \ text {5,5} \\ T_ {2} & = \ text {273,2} \ text {K} \ end {align *}

Газ будет иметь температуру \ (\ text {273,2} \) \ (\ text {K} \).

Проверьте свой ответ

Закон Чарльза гласит, что температура прямо пропорциональна объему. В этом примере объем уменьшается, поэтому температура должна снизиться. Наш ответ дает более низкую конечную температуру, чем начальная температура, и поэтому является правильным.

Рабочий пример 4: закон Карла

Хлорид аммония и гидроксид кальция могут вступить в реакцию. Аммиак, который выделяется в результате реакции, собирается в газовый шприц (шприц с очень небольшим трением, так что поршень может свободно перемещаться) и герметично закрывается.Этому газу позволяют нагреться до комнатной температуры, которая равна \ (\ text {20} \) \ (\ text {℃} \). Объем аммиака равен \ (\ text {122} \) \ (\ text {mL} \). Теперь он помещен в водяную баню, установленную в \ (\ text {32} \) \ (\ text {℃} \). Каким будет показание объема после того, как шприц оставлен в ванне на \ (\ text {1} \) \ (\ text {hour} \) (предположим, что поршень движется совершенно свободно)? (Оставив шприц на это время, мы можем быть уверены, что образец газа имеет более высокую температуру.)

Запишите всю информацию, которую вы знаете о газе.

\ begin {align *} V_ {1} & = \ text {122} \ text {mL} \\ V_ {2} & =? \\ T_ {1} & = \ text {20} \ text {℃} \\ T_ {2} & = \ text {7} \ text {℃} \ end {align *}

При необходимости преобразуйте известные значения в единицы СИ.

Здесь температуру нужно перевести в Кельвина, следовательно:

\ begin {align *} T_ {1} & = \ text {20} + \ text {273} = \ text {293} \ text {K} \\ T_ {2} & = \ text {32} + \ text {273} = \ text {305} \ text {K} \ end {align *}

Выберите соответствующее уравнение закона газа, которое позволит вам вычислить неизвестную переменную.

Давление поддерживается постоянным при изменении объема и температуры. Количество газа также остается постоянным, поэтому мы можем использовать закон Чарльза: \ [\ frac {V_ {2}} {T_ {2}} = \ frac {V_ {1}} {T_ {1}} \]

Подставьте известные значения в уравнение. Рассчитайте неизвестную переменную.

\ begin {align *} \ frac {V_ {2}} {\ text {305}} & = \ frac {\ text {122}} {\ text {293}} \\ \ frac {V_ {2}} {\ text {305}} & = \ text {0,416} \ ldots \\ V_ {2} & = \ text {127} \ text {mL} \ end {align *}

Значение объема на шприце будет \ (\ text {127} \) \ (\ text {mL} \) после того, как шприц будет оставлен в водяной бане на один час.

Проверьте свой ответ

Закон Чарльза гласит, что температура прямо пропорциональна объему. В этом примере температура увеличивается, поэтому объем должен увеличиваться. Наш ответ дает более высокий конечный объем, чем исходный, и поэтому является правильным.

Обратите внимание, что здесь температура должна быть преобразована в Кельвин, поскольку изменение градусов Цельсия включает сложение, а не умножение на фиксированный коэффициент преобразования (как в случае с давлением и объемом.)

Вы справитесь! Позвольте нам помочь вам учиться с умом для достижения ваших целей. Siyavula Practice направит вас в удобном для вас темпе, когда вы задаете вопросы в Интернете.

Зарегистрируйтесь, чтобы улучшить свои оценки

Закон Чарльза

Упражнение 7.3

Нарисуйте график, показывающий взаимосвязь между температурой и объемом.

Опишите отношения, которые вы наблюдаете.

По мере увеличения объема увеличивается и температура.

Если вы экстраполируете график (другими словами, удлините линию графика, даже если у вас нет точных точек данных), при какой температуре он пересекает ось \ (x \)?

График пересекает ось \ (x \) в точке \ (- \ text {273} \) \ (\ text {℃} \)

Что примечательного в этой температуре?

Эта точка соответствует абсолютному нулю. {3} $} \).{3} $} \]

\ begin {align *} \ frac {V_ {1}} {T_ {1}} & = \ frac {V_ {2}} {T_ {2}} \\ \ frac {\ text {1,2}} {\ text {340}} & = \ frac {\ text {0,2}} {T_ {2}} \\ \ text {0,003529} \ ldots & = \ frac {\ text {0,2}} {T_ {2}} \\ (\ text {0,003529} \ ldots) T_ {2} & = \ text {0,2} \\ & = \ text {56,67} \ text {K} \ end {align *}

Объясните, что произошло бы, если бы вы проверяли закон Чарльза и позволили газу уйти.

Если часть газа улетучится, объем газа уменьшится. Это привело бы к снижению температуры. Таким образом, конечная температура, которую вы прочитаете, будет ниже, чем должна быть. Это приведет к неправильному выводу, что закон Чарльза неверен.

Зависимость давления от температуры (ESBNY)

Давление газа прямо пропорционально его температуре, если объем поддерживается постоянным (рисунок 7.7). Напомним, что с увеличением температуры газа увеличивается кинетическая энергия частиц в газе.Это заставляет частицы в газе двигаться быстрее и чаще сталкиваться друг с другом и со стенкой контейнера. Поскольку давление является мерой этих столкновений, давление газа увеличивается с повышением температуры. Давление газа уменьшится, если его температура снизится.

Вы можете увидеть этот закон, называемый законом Гей-Люссака или законом Амонтона. Многие ученые работали над одними и теми же проблемами в одно и то же время, и часто бывает трудно узнать, кто на самом деле открыл тот или иной закон.

Рисунок 7.7: Связь между температурой и давлением газа.

Так же, как мы сделали для других законов о газах, мы можем описать взаимосвязь между температурой и давлением, используя следующие символы:

\ (Т ∝ p \),

следовательно:

\ (р = кТ \)

Переставляя это, мы получаем:

\ [\ frac {p} {T} = k \]

и что, при условии, что количество газа останется прежним (и объем также останется прежним):

\ [\ в коробке {\ dfrac {p_ {1}} {T_ {1}} = \ frac {p_ {2}} {T_ {2}}} \]

Рабочий пример 5: Зависимость давления от температуры

При температуре \ (\ text {298} \) \ (\ text {K} \) , определенное количество кислорода (\ (\ text {O} _ {2} \)) газа имеет давление \ (\ text {0,4} \) \ (\ text {atm} \).Какой температуры будет газ, если его давление увеличится до \ (\ text {0,7} \) \ (\ text {atm} \)?

Запишите всю информацию, которую вы знаете о газе.

\ begin {align *} T_ {1} & = \ text {298} \ text {K} \\ T_ {2} & =? \\ p_ {1} & = \ text {0,4} \ text {atm} \\ p_ {2} & = \ text {0,7} \ text {atm} \ end {align *}

При необходимости преобразуйте известные значения в единицы СИ.

Температура уже в Кельвинах. Нам не нужно переводить давление в паскали.

Выберите соответствующее уравнение закона газа, которое позволит вам вычислить неизвестную переменную.

Объем поддерживается постоянным при изменении давления и температуры. Количество газа также остается постоянным, поэтому мы можем использовать соотношение давления и температуры: \ [\ frac {p_ {1}} {T_ {1}} = \ frac {p_ {2}} {T_ {2}} \]

Подставьте известные значения в уравнение.Рассчитайте неизвестную переменную.

\ begin {align *} \ frac {\ text {0,4}} {\ text {298}} & = \ frac {\ text {0,7}} {T_ {2}} \\ \ text {0,0013} \ ldots & = \ frac {\ text {0,7}} {T_ {2}} \\ (\ text {0,0013} \ ldots) T_ {2} & = \ text {0,7} \\ T_ {2} & = \ text {521,5} \ text {K} \ end {align *}

Температура будет \ (\ text {521,5} \) \ (\ text {K} \).

Проверьте свой ответ

Зависимость давления от температуры утверждает, что давление прямо пропорционально температуре.В этом примере давление увеличивается, поэтому температура должна увеличиваться. Наш ответ дает более высокую конечную температуру, чем начальная температура, и поэтому является правильным.

Рабочий пример 6: Зависимость давления от температуры

Фиксированный объем газа окиси углерода (\ (\ text {CO} \)) имеет температуру \ (\ text {32} \) \ (\ text {℃} \) и давление \ (\ text { 680} \) \ (\ text {Pa} \). Если температура снизится до \ (\ text {15} \) \ (\ text {℃} \), какое будет давление?

Запишите всю информацию, которую вы знаете о газе.

\ begin {align *} T_ {1} & = \ text {32} \ text {℃} \\ T_ {2} & = \ text {15} \ text {℃} \\ p_ {1} & = \ text {680} \ text {Pa} \\ p_ {2} & =? \ end {align *}

При необходимости преобразуйте известные значения в единицы СИ.

Нам нужно преобразовать данные температуры в температуру Кельвина. \ begin {align *} T_ {1} & = \ text {32} + \ text {273} \\ & = \ text {305} \ text {K} \\ T_ {2} & = \ text {15} + \ text {273} \\ & = \ текст {288} \ текст {K} \ end {align *}

Выберите соответствующее уравнение закона газа, которое позволит вам вычислить неизвестную переменную.

\ [\ frac {p_ {2}} {T_ {2}} = \ frac {p_ {1}} {T_ {1}} \]

Подставьте известные значения в уравнение. Рассчитайте неизвестную переменную.

\ begin {align *} \ frac {p_ {2}} {\ text {288}} & = \ frac {\ text {680}} {\ text {305}} \\ \ frac {p_ {2}} {\ text {288}} & = \ text {2,2295} \ ldots \\ p_ {2} & = \ text {642,1} \ text {Па} \ end {align *}

Давление будет \ (\ text {642,1} \) \ (\ text {Pa} \).

Проверьте свой ответ

Зависимость давления от температуры утверждает, что давление прямо пропорционально температуре.В этом примере температура снижается, поэтому давление должно уменьшаться. Наш ответ дает более низкое конечное давление, чем начальное, и поэтому является правильным.

Зависимость давления от температуры

Упражнение 7.4

Перевести все температурные данные в Кельвины.

Давление (\ (\ text {atm} \))

Температура (\ (\ text {℃} \))

Температура (\ (\ text {K}) \))

\ (\ text {1,0} \)

\ (\ text {20} \)

\ (\ text {293} \)

\ (\ text {1,2} \)

\ (\ text {78,6} \)

\ (\ text {351,6} \)

\ (\ text {1,4} \)

\ (\ text {137,2} \)

\ (\ text {410,2} \)

\ (\ текст {1,6} \)

\ (\ text {195,8} \)

\ (\ text {468,8} \)

\ (\ text {1 , 8} \)

\ (\ text {254,4} \)

\ (\ text {527,4} \)

\ (\ text {2,0} \)

\ (\ text {313} \)

\ (\ text {586} \)

Нарисуйте график, показывающий взаимосвязь между температурой и давлением.

Опишите отношения, которые вы наблюдаете.

Есть линейная зависимость. Давление и температура прямо пропорциональны. График представляет собой прямую линию.

Баллон, содержащий газообразный метан, поддерживается при температуре \ (\ text {15} \) \ (\ text {℃} \) и оказывает давление \ (\ text {7} \) \ (\ text { атм} \). Если температура в цилиндре увеличивается до \ (\ text {25} \) \ (\ text {℃} \), какое давление теперь оказывает газ?

Нам нужно преобразовать данные температуры в температуру Кельвина.

\ begin {align *} T_ {1} & = \ text {15} + \ text {273} \\ & = \ text {288} \ text {K} \\ T_ {2} & = \ text {25} + \ text {273} \\ & = \ текст {298} \ текст {K} \ end {выровнять *} \ begin {align *} \ frac {p_ {2}} {T_ {2}} & = \ frac {p_ {1}} {T_ {1}} \\ \ frac {p_ {2}} {\ text {298}} & = \ frac {\ text {7}} {\ text {288}} \\ \ frac {p_ {2}} {\ text {298}} & = \ text {0,024} \ ldots \\ p_ {2} & = \ text {7,24} \ text {atm} \ end {выровнять *}

Давление будет \ (\ text {7,24} \) \ (\ text {atm} \).

Баллон с пропаном при температуре \ (\ text {20} \) \ (\ text {℃} \) оказывает давление \ (\ text {8} \) \ (\ text {atm} \) . Когда цилиндр помещен на солнечный свет, его температура повышается до \ (\ text {25} \) \ (\ text {℃} \). Какое давление газа внутри баллона при этой температуре?

Сначала переводим температуру в Кельвин:

\ begin {align *} T_ {1} & = \ text {20} + \ text {273} = \ text {293} \ text {K} \\ T_ {2} & = \ text {25} + \ text {273} = \ text {298} \ text {K} \ end {выровнять *}

И тогда мы можем найти давление:

\ begin {align *} \ frac {p_ {2}} {T_ {2}} & = \ frac {p_ {1}} {T_ {1}} \\ \ frac {p_ {2}} {\ text {298}} & = \ frac {\ text {8}} {\ text {293}} \\ \ frac {p_ {2}} {\ text {298}} & = \ text {0,0273} \ ldots \\ p_ {2} & = \ text {8,14} \ text {atm} \ end {выровнять *}

Баллончик с лаком для волос — это баллончик с газом под высоким давлением.На банке написано следующее предупреждение: «Не ставьте рядом с открытым огнем. Не выбрасывать в огонь. Беречь от тепла ». Используйте то, что вы знаете о давлении и температуре газов, чтобы объяснить, почему несоблюдение этого предупреждения опасно.

Давление газа прямо пропорционально его температуре при фиксированном объеме. Это соотношение сохраняется, поскольку объем газа в аэрозольном баллоне не меняется. Если повысить температуру банки, поместив ее рядом с открытым пламенем или в жаркое место, давление возрастет.Это может привести к взрыву баллона. То же самое верно, если вы выбрасываете в огонь.

Баллон с газообразным ацетиленом поддерживается при температуре \ (\ text {291} \) \ (\ text {K} \). Давление в цилиндре равно \ (\ text {5} \) \ (\ text {atm} \). Этот цилиндр может выдержать давление \ (\ text {8} \) \ (\ text {atm} \), прежде чем он взорвется. При какой максимальной температуре можно безопасно хранить баллон?

\ begin {align *} \ frac {p_ {1}} {T_ {1}} & = \ frac {p_ {2}} {T_ {2}} \\ \ frac {\ text {5}} {\ text {291}} & = \ frac {\ text {8}} {T_ {2}} \\ \ text {0,017} \ ldots & = \ frac {\ text {8}} {T_ {2}} \\ (\ text {0,017} \ ldots) T_ {2} & = \ text {8} \\ T_ {2} & = \ text {465,6} \ text {K} \ end {выровнять *}

Общее уравнение газа (ESBNZ)

Все газовые законы, которые мы описали до сих пор, основаны на том факте, что количество газа и одна другая переменная (температура, давление или объем) остаются постоянными.Поскольку в большинстве случаев это маловероятно, полезно объединить зависимости в одно уравнение. Мы будем использовать закон Бойля и зависимость давления от температуры, чтобы составить общее уравнение газа.

Закон Бойля: \ (p ∝ \ dfrac {1} {V} \) (постоянная T)

Другими словами, давление обратно пропорционально объему при постоянной температуре.

Зависимость давления от температуры: \ (p ∝ T \) (постоянная V)

Другими словами, давление также прямо пропорционально температуре при постоянном объеме.

Если мы теперь изменим и объем, и температуру, две пропорциональности все еще сохранятся, но будут равны другой константе пропорциональности.

Итак, мы можем объединить эти отношения, чтобы получить:

\ [p ∝ \ frac {T} {V} \]

Обратите внимание, это говорит о том, что давление по-прежнему прямо пропорционально температуре и обратно пропорционально объему.

Если мы введем коэффициент пропорциональности k, мы получим:

\ [p = k \ frac {T} {V} \]

или, переставляя уравнение:

\ [pV = kT \]

Мы также можем переписать это соотношение следующим образом:

\ [\ frac {pV} {T} = k \]

При условии, что масса газа остается прежней, мы также можем сказать, что:

\ [\ в коробке {\ frac {p_ {1} V_ {1}} {T_ {1}} = \ frac {p_ {2} V_ {2}} {T_ {2}}} \]

В приведенном выше уравнении нижние индексы \ (\ text {1} \) и \ (\ text {2} \) относятся к двум показаниям давления и объема для одной и той же массы газа в разных условиях.{3} $} \\ p_ {1} & = \ text {170} \ text {кПа} \\ p_ {2} & =? \ end {align *}

При необходимости преобразуйте известные значения в единицы СИ.

Нам нужно преобразовать данные температуры в температуру Кельвина. \ begin {align *} T_ {1} & = \ text {16} + \ text {273} \\ & = \ text {289} \ text {K} \\ T_ {2} & = \ text {40} + \ text {273} \\ & = \ текст {313} \ текст {K} \ end {align *}

Выберите соответствующее уравнение закона газа, которое позволит вам вычислить неизвестную переменную.

Температура, давление и объем меняются, поэтому мы должны использовать общее уравнение газа: \ [\ frac {p_ {2} V_ {2}} {T_ {2}} = \ frac {p_ {1} V_ {1}} {T_ {1}} \]

Подставьте известные значения в уравнение. {3} $} \\ V_ {2} & =? \ end {align *}

При необходимости преобразуйте известные значения в единицы СИ.

Здесь температуру нужно перевести в Кельвина, следовательно:

\ begin {align *} T_ {1} & = \ text {15} + \ text {273} = \ text {288} \ text {K} \\ T_ {2} & = \ text {32} + \ text {273} = \ text {305} \ text {K} \ end {align *}

Выберите соответствующее уравнение закона газа, которое позволит вам вычислить неизвестную переменную.

Мы используем общее уравнение газа:

\ [\ frac {p_ {2} V_ {2}} {T_ {2}} = \ frac {p_ {1} V_ {1}} {T_ {1}} \]

Подставьте известные значения в уравнение.{3} $} \\ p_ {1} & = \ text {5} \ text {atm} \\ p_ {2} & = \ text {5,2} \ text {atm} \ end {align *}

При необходимости преобразуйте известные значения в единицы СИ.

Температурные данные уже указаны в градусах Кельвина. Все остальные значения указаны в тех же единицах измерения.

Выберите соответствующее уравнение закона газа, которое позволит вам вычислить неизвестную переменную.

Поскольку объем, давление и температура меняются, мы должны использовать общее уравнение газа:

\ [\ frac {p_ {1} V_ {1}} {T_ {1}} = \ frac {p_ {2} V_ {2}} {T_ {2}} \]

Подставьте известные значения в уравнение.Рассчитайте неизвестную переменную.

\ begin {align *} \ frac {(\ text {5}) (\ text {4})} {\ text {298}} & = \ frac {(\ text {5,2}) (\ text {3,7})} { Т_ {2}} \\ \ text {0,067} \ ldots & = \ frac {\ text {19,24}} {T_ {2}} \\ (\ text {0,067} \ ldots) T_ {2} & = \ text {19,24} \\ T_ {2} & = \ text {286,7} \ text {K} \ end {align *}

Температура будет \ (\ text {286,7} \) \ (\ text {K} \).

Общее уравнение газа

Упражнение 7.5

Замкнутая газовая система изначально имеет объем \ (\ text {8} \) \ (\ text {L} \) и температуру \ (\ text {100} \) \ (\ text {℃} \) . Давление газа неизвестно. Если температура газа снижается до \ (\ text {50} \) \ (\ text {℃} \), газ занимает объем \ (\ text {5} \) \ (\ text {L} \ ), а давление газа равно \ (\ text {1,2} \) \ (\ text {atm} \). Какое было начальное давление газа?

Сначала переводим температуру в Кельвин:

\ begin {align *} T_ {1} & = \ text {100} + \ text {273} = \ text {373} \ text {K} \\ T_ {2} & = \ text {50} + \ text {273} = \ text {323} \ text {K} \ end {выровнять *}

Теперь мы можем использовать уравнение газа:

\ begin {align *} \ frac {p_ {1} V_ {1}} {T_ {1}} & = \ frac {p_ {2} V_ {2}} {T_ {2}} \\ \ frac {(p_ {1}) (\ text {8})} {\ text {373}} & = \ frac {(\ text {1,2}) (\ text {5})} {\ text { 323}} \\ \ frac {(p_ {1}) (\ text {8})} {\ text {373}} & = \ text {0,0185} \ ldots \\ (p_ {1}) (\ text {8}) & = \ text {6,928} \ ldots \\ p_ {1} & = \ text {0,87} \ text {atm} \ end {выровнять *}

Воздушный шар заполнен газообразным гелием в точке \ (\ text {27} \) \ (\ text {℃} \) и давлении \ (\ text {1,0} \) \ (\ text {atm} \ ).Когда воздушный шар поднимается, объем воздушного шара увеличивается в \ (\ text {1,6} \) раз, а температура снижается до \ (\ text {15} \) \ (\ text {℃} \). Каково конечное давление газа (при условии, что ни один газ не улетучился)?

Сначала переводим температуру в Кельвин:

\ begin {align *} T_ {1} & = \ text {27} + \ text {273} = \ text {300} \ text {K} \\ T_ {2} & = \ text {15} + \ text {273} = \ text {288} \ text {K} \ end {выровнять *}

Пусть начальный объем будет \ (V_ {1} \), а конечный объем будет \ (\ text {1,6} V_ {1} \)

Теперь мы можем использовать уравнение газа:

\ begin {align *} \ frac {p_ {2} V_ {2}} {T_ {2}} & = \ frac {p_ {1} V_ {1}} {T_ {1}} \\ \ frac {p_ {2} (\ text {1,6} V_ {1})} {\ text {288}} & = \ frac {(\ text {1}) V_ {1}} {\ text {300 }} \\ \ text {480} V_ {1} p_ {2} & = \ text {288} V_ {1} \\ p_ {2} & = \ text {0,6} \ text {atm} \ end {выровнять *}

\ (\ text {25} \) \ (\ text {cm $ ^ {3} $} \) газа в \ (\ text {1} \) \ (\ text {atm} \) имеет температуру \ (\ text {25} \) \ (\ text {℃} \).{3} $} \) температура газа увеличивается до \ (\ text {28} \) \ (\ text {℃} \). Рассчитайте конечное давление газа.

Сначала переводим температуру в Кельвин:

\ begin {align *} T_ {1} & = \ text {25} + \ text {273} = \ text {298} \ text {K} \\ T_ {2} & = \ text {28} + \ text {273} = \ text {301} \ text {K} \ end {выровнять *}

Теперь мы можем использовать уравнение газа:

\ begin {align *} \ frac {p_ {2} V_ {2}} {T_ {2}} & = \ frac {p_ {1} V_ {1}} {T_ {1}} \\ \ frac {(p_ {2}) (\ text {20})} {\ text {301}} & = \ frac {(\ text {1}) (\ text {25})} {\ text {298} } \\ \ frac {(p_ {2}) (\ text {20})} {\ text {301}} & = \ text {0,08389} \ ldots \\ (p_ {2}) (\ text {20}) & = \ text {25,516} \ ldots \\ p_ {1} & = \ text {1,26} \ text {atm} \ end {выровнять *}

Уравнение идеального газа (ESBP2)

В начале 1800-х годов Амедео Авогадро отмечал, что если у вас есть образцы разных газов, одного и того же объема, при фиксированной температуре и давлении, то образцы должны содержать одинаковое количество свободно движущихся частиц (т.е. атомы или молекулы).

Закон Авогадро

Одинаковые объемы газов при одинаковой температуре и давлении содержат одинаковое количество молекул.

Как вы помните из предыдущего раздела, мы объединили различные уравнения газового закона, чтобы получить одно , которое включает температуру, объем и давление. В этом уравнении

\ [\ dfrac {pV} {T} = k \]

значение k различно для разных масс газа.{3} $} \). Поэтому, когда вы используете уравнение идеального газа, вы должны использовать единицы СИ, чтобы получить правильный ответ.

Если теперь распространить этот результат на любое количество молей газа, мы получим следующее:

\ [\ frac {pV} {T} = nR \]

где \ (n \) — количество молей газа. {3} $} \\ p & = \ text {208 182,56} \ text {Па} \ end {align *}

Давление будет \ (\ text {208 182,56} \) \ (\ text {Pa} \) или \ (\ text {208,2} \) \ (\ text {kPa} \).

Рабочий пример 11: Уравнение идеального газа

Углекислый газ \ ((\ text {CO} _ {2}) \) образуется в результате реакции между карбонатом кальция и соляной кислотой. Произведенный газ собирается в контейнере неизвестного объема. Давление газа равно \ (\ text {105} \) \ (\ text {kPa} \) при температуре \ (\ text {20} \) \ (\ text {℃} \). Если количество собранных молей газа равно \ (\ text {0,86} \) \ (\ text {mol} \), каков объем?

Запишите всю информацию, которую вы знаете о газе.{-1} $} \ end {align *}

При необходимости преобразуйте известные значения в единицы СИ.

Нам нужно преобразовать температуру в Кельвин, а давление в \ (\ text {Па} \): \ begin {align *} p & = \ text {105} \ times \ text {1 000} = \ text {105 000} \ text {Pa} \\ T & = \ text {20} + \ text {273} = \ text {293} \ text {K} \ end {align *}

Выберите соответствующее уравнение закона газа, которое позволит вам вычислить неизвестную переменную.{3} $} \\ p & = \ text {195,89} \ times \ text {1 000} \\ & = \ text {195 890} \ text {Па} \ end {align *}

Выберите соответствующее уравнение закона газа, которое позволит вам вычислить неизвестную переменную.

\ [pV = nRT \]

Подставьте известные значения в уравнение. Рассчитайте неизвестную переменную.

\ begin {align *} (\ text {195 890}) (\ text {0,025}) & = (\ text {2}) (\ text {8,314}) T \\ \ text {4 897,25} & = \ text {16,628} (T) \\ Т & = \ текст {294,52} \ текст {К} \ end {align *}

Температура \ (\ text {294,52} \) \ (\ text {K} \).

Рабочий пример 13: Уравнение идеального газа

Рассчитайте количество молей частиц воздуха в классе длиной \ (\ text {10} \) \ (\ text {m} \), шириной \ (\ text {7} \) \ (\ text { m} \) и высоту \ (\ text {2} \) \ (\ text {m} \) в день, когда температура равна \ (\ text {23} \) \ (\ text {℃} \ ) и давление воздуха \ (\ text {98} \) \ (\ text {kPa} \) .

Рассчитать объем воздуха в классе

Класс представляет собой прямоугольную призму (вспомните оценку \ (\ text {10} \) математики по замерам).{-1} $} \\ T & = \ text {23} \ text {℃} \ end {align *}

При необходимости преобразуйте известные значения в единицы СИ.

Мы должны преобразовать температуру в \ (\ text {K} \) и давление в \ (\ text {Pa} \): \ begin {align *} T & = \ text {25} + \ text {273} = \ text {298} \ text {K} \\ p & = \ text {98} \ times \ text {1 000} = \ text {98 000} \ text {Па} \ end {align *}

Выберите соответствующее уравнение закона газа, которое позволит вам вычислить неизвестную переменную.

\ [pV = nRT \]

Подставьте известные значения в уравнение. Рассчитайте неизвестную переменную.

\ begin {align *} (\ text {98 000}) (\ text {140}) & = n (\ text {8,314}) (\ text {298}) \\ \ text {13 720 000} & = \ text {2 477,572} (n) \\ n & = \ text {5 537,7} \ text {mol} \ end {align *}

Количество родинок в классе: \ (\ text {5 537,7} \) \ (\ text {mol} \).

Уравнение идеального газа

Упражнение 7.6

Неизвестный газ имеет давление \ (\ text {0,9} \) \ (\ text {atm} \), температуру \ (\ text {120} \) \ (\ text {℃} \) и количество родинок \ (\ text {0,28} \) \ (\ text {mol} \). Какой объем пробы?

Сначала преобразуйте все единицы в единицы СИ:

\ begin {align *} T & = \ text {273} + \ text {120} = \ text {393} \ text {K} \\ V & =? \\ p & = \ frac {\ text {101 325}} {\ text {0,9}} = \ text {112 583,33} \ text {Па} \ end {выровнять *}

Теперь мы можем использовать уравнение идеального газа, чтобы найти объем газа:

\ begin {align *} pV & = nRT \\ (\ text {112 583,33}) V & = (\ text {0,28}) (\ text {8,314}) (\ text {393}) \\ \ text {112 583,33} V & = \ text {895,7256} \\ V & = \ text {0,008} \ text {m $ ^ {3} $} \\ V & = \ текст {8} \ text {dm $ ^ {3} $} \ end {выровнять *}

\ (\ text {6} \) \ (\ text {g} \) хлора \ ((\ text {Cl} _ {2}) \) занимает объем \ (\ text {0,002} \) \ (\ text {m $ ^ {3} $} \) при температуре \ (\ text {26} \) \ (\ text {℃} \).Какое давление газа в этих условиях?

Сначала найдите количество молей газообразного хлора:

\ begin {align *} n & = \ frac {m} {M} \\ & = \ frac {\ text {6}} {\ text {70}} \\ & = \ текст {0,0857} \ текст {мол} \ end {выровнять *}

Затем преобразовать все единицы в единицы СИ:

\ [T = \ text {273} + \ text {26} = \ text {299} \ text {K} \]

Теперь мы можем использовать уравнение идеального газа, чтобы найти давление:

\ begin {align *} pV & = nRT \\ (p) (\ text {0,002}) & = (\ text {0,0857}) (\ text {8,314}) (\ text {299}) \\ (p) (\ text {0,002}) & = \ text {213,075} \ ldots \\ & = \ text {106 537,9714} \ text {Па} \\ & = \ text {106,54} \ text {кПа} \ end {выровнять *}

Средняя пара легких человека содержит примерно \ (\ text {3,5} \) \ (\ text {L} \) воздуха после вдоха и примерно \ (\ text {3,0} \) \ (\ text {L} \) после выдоха.{3} $} \ end {выровнять *}

Перевести все единицы в единицы СИ:

\ begin {align *} T & = \ text {273} + \ text {37} = \ text {310} \ text {K} \\ p & = \ text {101 325} \ text {Па} \ end {выровнять *}

Теперь мы можем использовать уравнение идеального газа, чтобы найти количество молей:

\ begin {align *} pV & = nRT \\ (\ text {101 325}) (\ text {0,0005}) & = (n) (\ text {8,314}) (\ text {310}) \\ \ text {50,6625} & = (n) (\ text {2 577,34}) \\ & = \ текст {0,02} \ текст {моль} \ end {выровнять *}

Определите \ (\ text {2} \) ошибки, которые учащийся сделал в вычислениях. {3} $} \) в \ (\ text {25} \) \ (\ text {℃} \) и \ (\ text {99,3} \) \ (\ text {kPa} \).{3} $} \\ T & = \ text {25} + \ text {273} = \ text {298} \ text {K} \ end {выровнять *} \ begin {align *} pV & = nRT \\ (\ text {99 300}) (\ text {0,065}) & = n (\ text {8,314}) (\ text {298}) \\ \ text {6 464,5} & = n (\ text {2 477,572}) \\ п & = \ текст {2,61} \ текст {мол} \ end {выровнять *}

Далее переводим количество молей в граммы:

\ begin {align *} n & = \ frac {m} {M} \\ \ text {2,61} & = \ frac {m} {\ text {28}} \\ м & = \ текст {73,1} \ текст {g} \ end {выровнять *}

Вышеупомянутая реакция выделяет тепло, которое приводит к повышению температуры подушки безопасности.Опишите в терминах кинетической теории газов, как давление в образце подушки безопасности изменится, если вообще изменится, когда температура газа вернется к \ (\ text {25} \) \ (\ text {℃} \).

При понижении температуры интенсивность столкновений со стенками подушки безопасности и между частицами уменьшается. Поэтому давление снижается.

12.2 Первый закон термодинамики: тепловая энергия и работа

Биология: биологическая термодинамика

Мы часто думаем о термодинамике, как о полезной для изобретения или тестирования оборудования, такого как двигатели или паровые турбины.Однако термодинамика также применима к живым системам, таким как наши собственные тела. Это составляет основу биологической термодинамики (рис. 12.7).

Рис. 12.7 (а) Первый закон термодинамики применим к метаболизму. Тепло, передаваемое из тела (Q), и работа, выполняемая телом (W), удаляют внутреннюю энергию, тогда как прием пищи заменяет ее. (Прием пищи можно рассматривать как работу, выполняемую телом.) (Б) Растения преобразуют часть лучистой энергии солнечного света в запасенную химическую энергию — процесс, называемый фотосинтез .

Сама жизнь зависит от биологической передачи энергии. Посредством фотосинтеза растения поглощают солнечную энергию солнца и используют эту энергию для преобразования углекислого газа и воды в глюкозу и кислород. Фотосинтез принимает одну форму энергии — свет — и преобразует ее в другую форму — химическую потенциальную энергию (глюкозу и другие углеводы).

Метаболизм человека — это преобразование пищи в энергию, выделяемую теплом, работу, выполняемую клетками организма, и накопленный жир.Метаболизм — интересный пример действия первого закона термодинамики. Прием пищи увеличивает внутреннюю энергию тела за счет добавления химической потенциальной энергии; это неромантичный взгляд на хороший буррито.

Организм усваивает всю пищу, которую мы потребляем. По сути, метаболизм — это процесс окисления, при котором высвобождается химическая потенциальная энергия пищи. Это означает, что питание осуществляется в форме работы. Упражнения помогают вам похудеть, поскольку они обеспечивают передачу энергии от вашего тела как за счет тепла, так и за счет работы, а также повышают уровень метаболизма, даже когда вы находитесь в состоянии покоя.

Биологическая термодинамика также включает изучение трансдукции между клетками и живыми организмами. Трансдукция — это процесс, при котором генетический материал — ДНК — передается от одной клетки к другой. Это часто происходит во время вирусной инфекции (например, гриппа), и именно так вирус распространяется, а именно путем переноса своего генетического материала на все большее количество ранее здоровых клеток. Когда инфицировано достаточное количество клеток, вы начинаете ощущать воздействие вируса (симптомы гриппа — мышечная слабость, кашель и заложенность носа).

Энергия передается вместе с генетическим материалом и, таким образом, подчиняется первому закону термодинамики. Энергия передается — а не создается и не уничтожается — в процессе. Когда с элементом выполняется работа или тепло передает энергию ячейке, внутренняя энергия ячейки увеличивается. Когда клетка работает или теряет тепло, ее внутренняя энергия уменьшается. Если количество работы, выполняемой ячейкой, такое же, как количество энергии, передаваемой теплом, или количество работы, выполняемой ячейкой, соответствует количеству энергии, передаваемой теплом, чистое изменение внутренней энергии не будет. .

Проверка захвата

Исходя из того, что вы знаете о теплопередаче и о первом законе термодинамики, нужно ли вам есть больше или меньше, чтобы поддерживать постоянный вес в холодную погоду? Объяснить, почему.

    еще
  1. ; поскольку в холодную погоду организм теряет больше энергии, потребность в еде увеличивается, чтобы поддерживать постоянный вес
  2. Еще
  3. ; употребление большего количества пищи означает накопление большего количества жира, что защитит организм от холодной погоды и уменьшит потерю энергии
  4. На
  5. меньше; поскольку в холодную погоду организм теряет меньше энергии, потребность в еде уменьшается, чтобы поддерживать постоянный вес
  6. На
  7. меньше; употребление меньшего количества пищи означает накопление меньшего количества жира, поэтому для сжигания жира потребуется меньше энергии, и в результате вес останется постоянным

газовых законов — гипертекст по физике

Обсуждение

введение

Газовые законы представляют собой набор интуитивно очевидных утверждений для большинства в сегодняшнем западном мире.Трудно поверить, что когда-нибудь их не понимали. И все же кто-то должен был заметить эти отношения и записать их. По этой причине многих студентов учат трем важнейшим законам о газе по именам их первооткрывателей. Однако, поскольку законы известны под разными именами в разных странах и (что более важно), так как я никогда не могу вспомнить, кому принадлежит какой закон, не обращаясь к примечаниям, я не буду следовать этому соглашению.

давление-объем (постоянная температура)

Что происходит с объемом газа при изменении давления на него.Давайте попробуем провести следующий эксперимент с оборудованием, которое может быть у вас на кухне.

Зефир в кухонном вакуумном насосе. Объем зефира увеличивается по мере уменьшения давления на него — и наоборот. Проведите эксперимент дома. Сделай это !

Зефир представляет собой смесь сахара, воздуха и желатина. Сахар делает их сладкими, воздух — пушистыми, а желатин — эластичными.Зефир — это замороженная пена, по объему она в основном воздушная. При помещении в вакуумный насос они расширяются при понижении давления. Сломайте пломбу на контейнере, и они сядут во время восстановления нормального атмосферного давления. Поскольку вакуумный насос тянет зефир достаточно сильно, чтобы лопнуть некоторые из пузырьков воздуха, они на самом деле немного меньше и более сморщиваются в конце этого эксперимента. Это иллюстрирует фундаментальное, но важное свойство газов. Давление газа обратно пропорционально его объему при постоянной температуре.Символически…

P 1 ( T постоянная)
В

или

P 1 V 1 = P 2 V 2 = постоянная

Эта корреляция была независимо обнаружена Робертом Бойлом (1627–1691) из Ирландии в 1662 году и Эдме Мариоттом (1620–1684) из Франции в 1676 году.В Великобритании, Америке, Австралии, Вест-Индии и других остатках Британской империи он называется законом Бойля , тогда как в континентальной Европе и других странах он называется законом Мариотта .

Мариотт добавил важное положение о том, что температура остается постоянной. Бойль не упомянул об этом, но данные, которые он использовал для вывода своего закона, скорее всего, были собраны в период, когда температура не претерпевала каких-либо значительных изменений. Поскольку газ должен находиться в тепловом равновесии с окружающей средой (или каким-либо другим тепловым резервуаром) для поддержания равномерной температуры, соотношение давление-объем обычно применяется только к «медленным» процессам.Показанный выше эксперимент с зефиром и вакуумом является примером «медленного» процесса. Давление снижается достаточно медленно, чтобы тепло окружающей среды могло поддерживать банку и ее содержимое почти при комнатной температуре. Такое превращение, которое происходит без изменения температуры, называется изотермическим .

Прокачка велосипедной шины ручным насосом — это пример «быстрого» процесса. Работа, выполняемая при нажатии на поршень, трансформируется в увеличение внутренней энергии (и, следовательно, повышение температуры) молекул воздуха внутри насоса.Люди, знакомые с ручными велосипедными насосами, подтвердят, что после использования они становятся горячими. Точно так же, когда газу позволяют расширяться в область пониженного давления, он действительно воздействует на окружающую среду. Энергия для выполнения этой работы исходит от внутренней энергии газа, поэтому температура газа падает. Вы можете испытать это сами, не прибегая к помощи каких-либо устройств, кроме рта. Сожмите губы так, чтобы у рта было только крошечное отверстие наружу, и дуйте с силой. Воздух, выходящий из вашего рта, будет довольно прохладным, несмотря на то, что он исходит из ядра вашего тела, которое обычно довольно горячее (около 37 ° C).Во время «быстрого» процесса, подобного только что описанному, давление и объем изменяются так быстро, что у тепла не хватает времени, чтобы войти в газ или выйти из него, чтобы поддерживать постоянную температуру. Такое преобразование, которое происходит без какого-либо теплового потока, называется адиабатическим .

объем-температура (постоянное давление)

Что происходит с объемом газа при изменении его температуры? Попробуем еще один кухонный эксперимент.

Хлебное тесто до и после выпечки.Повышение температуры хлебного теста увеличивает его объем. Проведите эксперимент дома. Сделай это !
[тесто] [хлеб]

Хлеб изготавливается из пшеничной муки, воды, дрожжей и небольшого количества сахара. Дрожжи — это крошечные микроорганизмы. Вполне возможно, что это первые одомашненные животные, и, как и собаки и лошади, дрожжи были выведены для разных целей. Так же, как у нас есть сторожевые собаки, болонки и охотничьи собаки; упряжные лошади, скаковые лошади и боевые лошади; у нас также есть пивные дрожжи, шампанские дрожжи и хлебные дрожжи.Хлебные дрожжи были селективно выведены для употребления в пищу сахара и отрыжки углекислого газа (CO 2 ). Когда пшеничная мука и вода смешиваются вместе и замешиваются, молекулы белка измельчаются и растягиваются до тех пор, пока они не выровняются, чтобы образовать вещество, называемое глютеном, которое, как и жевательная резинка, является эластичным и пластичным. Дайте этой специальной матрице отстояться, и выделившийся из дрожжей CO 2 окажется в тысячах крошечных упругих эластичных карманов. По мере продолжения этого процесса эти крошечные карманы расширяются, в результате чего объем теста увеличивается или увеличивается в процессе, называемом расстойка .Теперь у нас есть пушистая мармеладная капля, готовая для духовки.

Пока там тесто снова расширяется, но его время это не из-за действия микроорганизмов (все они умирают около точки кипения воды). На этот раз дело в жаре, а точнее в температуре. Температура внутри хлебной печи примерно на 50% выше (в абсолютном выражении), чем температура снаружи. Точно так же испеченный хлеб, который выходит из хлебной печи, также примерно на 50% больше, чем тесто комнатной температуры, которое идет в него.Этот отечественный пример прекрасно иллюстрирует фундаментальное свойство газов. При постоянном давлении объем газа прямо пропорционален его температуре. Символически…

В T ( P постоянная)

Хотя это, без сомнения, известно и неформально понимается миллиардами пекарей с самого начала цивилизации, точное математическое соотношение было впервые обнаружено французским физиком Гийомом Амонтоном (1663–1705) в 1699 году. Эксперимент был повторен намного позже Жаком Шарлем (1746). –1823) в 1787 году и намного позже Жозефом Гей-Люссаком (1778–1850) в 1802 году.Чарльз не опубликовал свои открытия, но Гей-Люссак опубликовал. Чаще всего он называется законом Чарльза в британской сфере влияния и законом Гей-Люссака во Франции, но никогда законом Амонтона.

Изобарический процесс — это процесс, который происходит без каких-либо изменений давления .

Давайте вспомним, что означает, когда две величины прямо пропорциональны, такие как объем и температура. Нагрейте газ, и его объем расширится.Охладите его, и его объем уменьшится. Две величины изменяются в одном направлении. Более конкретно, увеличение одного приводит к пропорциональному увеличению другого, а уменьшение одного приводит к пропорциональному уменьшению другого. Например…

  • Удвоение абсолютной температуры воздуха в цилиндре двигателя увеличивает его объем вдвое.
  • Уменьшение вдвое абсолютной температуры воздуха в пакете с картофельными чипсами приведет к его уменьшению вдвое от первоначального объема.
  • Абсолютная температура хлебной печи в полтора раза выше комнатной. Таким образом, буханка выпеченного хлеба, выходящая из духовки, имеет на 50% больше объема, чем шарик теста, который в нее вошел.

Здесь где-то работает симметрия. Симметрия — это изменение одной величины, которое оставляет неизменной другую, более фундаментальную величину. Это что-то вроде умножения числителя и знаменателя дроби на одно и то же.

а

х

= a x = a
б х b x б

Нет, погоди, это именно так.Единственный способ, которым две величины могут изменяться прямо пропорционально, — это если их соотношение остается постоянным. Таким образом…

В 1 = В 2 = постоянная
Т 1 Т 2

давление-температура (постоянный объем)

Исправьте и это.

Давление газа прямо пропорционально его температуре при постоянном объеме.Символически…

P T ( V постоянная)

Изохорный процесс — это процесс, который происходит без каких-либо изменений в томе .

У этой связи на самом деле нет названия, но я слышал, что ее называют «законом давления» или (ошибочно) «законом Гей-Люссака».

Температура падает на 6 ° C на каждые 1000 м высоты.

п. 1 = п 2 = постоянная
Т 1 Т 2

абсолютная температура

В 1703 году Амонтонс заявил…?

Двойная комнатная температура, 293 K = 20 ° C, и вы получите 586 K = 313 ° C, а не 40 ° C.

полный закон идеального газа

Утверждения о пропорциональности сегодня не так популярны в 21 веке, как в 19 веке и ранее. Мы живем в эпоху, когда все решает уравнение. В этом фокусе есть хорошее и плохое. Уравнения передают много информации в нескольких символах, поэтому они так популярны, но они также являются костылем; устройство, используемое для поддержки слабого понимания и придания ему уверенности. Уравнения могут использоваться учащимся, не понимающим, для подделки компетенций.

«Я ввел числа в уравнение и получил правильный ответ. Поскольку у меня есть правильный ответ, я умен».

Опытный? Безусловно. Умная? Не обязательно.

Тем не менее, было бы неплохо иметь уравнение для тех случаев, когда все, что вам нужно, — это просто выполнить работу с минимумом хлопот.

Объедините три вместе.

P 1 V 1 = P 2 V 2 = постоянная
Т 1 Т 2

Есть два способа записать полное изложение закона идеального газа в виде уравнения…

Функциональная термодинамика

PV = nRT

где…

P = абсолютное давление
Т = абсолютная температура
В = том

и

n = количество родинок
R = газовая постоянная = 8.315 Дж / моль · K

статистическая термодинамика

PV = NkT

где…

P = абсолютное давление
Т = абсолютная температура
В = том

и

N = количество частиц
к = Постоянная Больцмана = 1.

Добавить комментарий

Ваш адрес email не будет опубликован. Обязательные поля помечены *

*

© 2011-2024 Компания "Кондиционеры"